Режим чтения
Скачать книгу

А что, если?.. Научные ответы на абсурдные гипотетические вопросы читать онлайн - Рэндалл Манро

А что, если?.. Научные ответы на абсурдные гипотетические вопросы

Рэндалл Манро

Эта книга – продолжение одного из самых знаменитых научно-популярных сайтов всех времен. Рэндалл Манро – инженер НАСА, ученый, художник и создатель невероятно популярного интернет-комикса xkcd.com – пытается найти серьезные ответы на самые невероятные вопросы, которые присылают ему посетители его сайта. Оказывается, о самых серьезных научных проблемах можно говорить легко и с большим юмором. Если вы любите науку, комиксы, Интернет и хорошую шутку – эта книга для вас.

В формате pdf A4 сохранен издательский дизайн.

Рэндалл Манро

А что, если?.. Научные ответы на абсурдные гипотетические вопросы

Randall Munroe

WHAT IF?

Serious Scientific Answers to Absurd Hypothetical Questions

Печатается с разрешения автора и литературных агентств The Gernert Company, Inc. и Andrew Nurnberg.

Copyright © by xkcd Inc., 2014

© К. С. Ромашко, перевод, 2014

© Издание на русском языке AST Publishers, 2015

* * *

Политех

Политехнический музей – национальный музей науки и техники, один из крупнейших научно-технических музеев мира.

Миссия музея – просвещение и популяризация научных и технических знаний:

• Мы верим, что миром движут любопытство и созидание.

• Мы открываем людям прошлое, настоящее и будущее науки.

• Мы создаем территорию просвещения, свободной мысли и смелого эксперимента.

Среди просветительских проектов музея – многочисленные выставки, знаменитый Лекторий, Научные лаборатории для детей, Фестиваль актуального научного кино, а также запущенная в 2014 году издательская программа, цель которой – поддержка самых качественных научно-популярных книг, отобранных экспертами музея и выпущенных в сотрудничестве с лучшими издательствами страны.

Вы держите в руках одну из этих книг.

Подробнее о музее и его проектах – на сайтеwww (http://www.polymus.ru/).polymus (http://www.polymus.ru/).ru (http://www.polymus.ru/)

Отказ от ответственности

Не пытайтесь повторить это дома. Автор этой книги всего лишь рисует комиксы в Интернете – он не эксперт по безопасности и не специалист в области здравоохранения. Кроме того, автор обожает смотреть, как что-нибудь горит или взрывается, и это его увлечение вряд ли всегда пойдет вам на пользу.

Издательство и автор не несут ответственности за разрушительные последствия, которые могут прямо или косвенно возникнуть в результате чтения этой книги.

Введение

Книга, которую вы держите в руках, – сборник ответов на гипотетические вопросы в жанре «А что, если?»

Эти вопросы приходят на мой сайт, где я не только веду нечто вроде рубрики советов для безумных ученых, но и выкладываю свой комикс xkcd.

Я не всегда рисовал комиксы. В свое время я изучал физику, а после окончания университета занимался робототехникой в НАСА, но в конце концов я ушел из этой конторы, чтобы полностью посвятить себя комиксам. Однако мой интерес к науке и математике никуда не делся и нашел себе новое применение – теперь я отвечаю на странные (а иногда вызывающие тревогу) вопросы в Интернете. В настоящей книге представлена подборка моих любимых ответов с сайта плюс немало новых вопросов, на которые я здесь отвечаю впервые.

Сколько себя помню, я пытался с помощью математики найти ответы на необычные вопросы. Когда мне было пять лет, моя мама записала один наш разговор и сохранила его. Когда она узнала, что я пишу эту книгу, она нашла записку и прислала ее мне. Вот этот диалог, дословно воспроизведенный с листочка бумаги, которому уже 25 лет.

Рэндалл: Каких предметов в нашем доме больше – мягких или твердых?

Джули: Не знаю.

Рэндалл: А во всем мире?

Джули: Понятия не имею.

Рэндалл: Ну, в каждом доме ведь есть три или четыре подушки, да?

Джули: Ну да.

Рэндалл: И в каждом доме есть где-то пятнадцать магнитиков, да?

Джули: Скорее всего.

Рэндалл: Значит, 15 плюс 3 или плюс 4… пусть будет 4… Получится 19, верно?

Джули: Верно.

Рэндалл: Значит, всего будет где-то три миллиарда мягких предметов и… пять миллиардов твердых. Ну и кто победил?

Джули: Похоже, твердые!

До сего дня я не имею ни малейшего представления, откуда я взял три и пять миллиардов. Очевидно, тогда я не вполне понимал, как работают цифры.

В математике я с годами набрался опыта, но причина, по которой я ею занимаюсь, та же, что была в пять лет: я хочу отвечать на вопросы.

Говорят, что глупых вопросов не бывает. Это неправда: думаю, например, что мой вопрос про твердые и мягкие предметы довольно глупый. Но оказывается, что если вы попытаетесь серьезно ответить на глупый вопрос, это может завести вас в довольно любопытные места.

Я все еще не знаю, каких предметов в мире больше – твердых или мягких, но, странствуя по этим любопытным местам, я узнал многое другое. Перед вами мои любимые эпизоды этих путешествий.

    Рэндалл Манро

Всемирный ураган

ВОПРОС: А что, если Земля и все, что на ней находится, внезапно перестанет вращаться, но при этом атмосфера сохранит свою скорость движения?

    – Эндрю Браун

ОТВЕТ: Практически все мы умрем. Но вот потом станет интересно.

В районе экватора поверхность Земли движется со скоростью 470 м/с относительно земной оси. Если бы Земля остановилась, а воздух продолжил бы движение, то в результате возник бы ураган со скоростью свыше 1000 км/ч.

Сильнее всего этот ураган бушевал бы на экваторе, но все, кто живет между 42° с. ш. и 42° ю. ш., а это примерно 85 % населения земного шара, внезапно оказались бы в зоне действия ветра, дующего со сверхзвуковой скоростью.

Вблизи земли такой ветер продержится всего несколько минут – трение о земную поверхность скоро его замедлит. Однако этих нескольких минут будет достаточно, чтобы обратить практически все построенное человеком в руины.

Вблизи полюсов ветра были бы слабее, но там нет городов, которые смогли бы избежать разрушения благодаря удаленности от экватора. Лонгйир, расположенный на норвежском острове Шпицберген, один из самых северных в мире поселков, и его тоже разрушили бы ветра, сравнимые по мощи с сильнейшими тропическими циклонами.

Если бы вы искали возможность переждать эту катастрофу, то некоторые шансы у вас появились бы в Хельсинки. Хотя столица Финляндии находится за 60° с. ш., ветер все равно сровняет ее с землей. Однако пласт скалистой породы, на котором стоит Хельсинки, пронизан сложной системой тоннелей, в которых прячутся подземный супермаркет, хоккейная площадка, бассейн и многое другое.

Не уцелело бы ни одного строения: даже достаточно прочные, чтобы устоять при таком ветре, оказались бы под угрозой. Как сказал однажды комик Рон Уайт, проблема не в том, что ветер дует, а в том, на что именно он дует.

Представьте себе, что вы сидите в огромном бункере, построенном из материала, вполне способного выдержать ветер, дующий со скоростью 1000 км/ч.

Бункер – это хорошо, и с вами все было бы в порядке… если бы у вас у одного был бункер. К сожалению, у вас, вероятно, есть соседи, и у соседей тоже есть бункеры, и если бункер соседа окажется менее устойчивым, чем ваш, вашим бункерам придется пережить столкновение на скорости 1000 км/ч.

Все человечество не погибнет[1 - В смысле,
Страница 2 из 15

не погибнет сразу (здесь и далее примечания автора, если не оговорено иное. – Прим. ред.).]. Лишь немногие из людей на поверхности выживут: летящие обломки уничтожат все, что не было защищено настолько, чтобы пережить как минимум ядерный взрыв. Тем не менее многие люди под землей остались бы в живых. Если бы вы находились в глубоком подвале, а еще лучше – в тоннеле метро, когда Земля остановилась, у вас был бы неплохой шанс выжить.

Будут и другие счастливчики. Несколько десятков ученых и сотрудников научной станции Амундсен – Скотт на Южном полюсе были бы в безопасности. Для них первым признаком беды стало бы то, что весь внешний мир в радиоэфире внезапно замолчал.

Эта таинственная тишина, вероятно, заинтриговала бы полярников на какое-то время, но в конце концов кто-нибудь из них обратил бы внимание на кое-что еще более необычное.

Воздух

Когда ветер у поверхности уляжется, начнутся еще более странные события.

Порыв ветра превратится в тепловую волну. Обычно кинетическая энергия порыва ветра не слишком велика, и ею можно пренебречь в расчетах, но мы же имеем дело не с каким-то заурядным ветерком! В общем, когда ветер стихнет, воздух станет нагреваться.

На поверхности Земли это приведет к аномальному повышению температуры, а в зонах влажного климата к бурям и грозам.

В то же время бушующие над океаном ураганы превратят верхний слой воды в водяную пыль. На какое-то время у океана вообще не станет поверхности, и будет невозможно понять, где заканчивается водяная пыль и начинается толща воды.

Океаны холодные. Средняя температура воды под тонким поверхностным слоем составляет всего 5°C. Буря подняла бы холодную воду из глубин. Обилие очень холодной водяной пыли в перегретом воздухе породило бы погоду, которой раньше на Земле никогда не бывало: одновременно ветер, туман, мелкий дождь и резкие перепады температуры.

Приток воды из глубины дал бы сильный толчок к развитию жизни, ведь свежие питательные вещества поднялись бы к верхним слоям. Но одновременно это привело бы к вымиранию множества рыб, крабов, морских черепах и других животных, неспособных дышать в поднявшейся из глубин воде с низким содержанием кислорода. А морским животным, которые дышат воздухом, например китам или дельфинам, было бы трудно выжить на постоянно меняющейся границе моря и воздуха.

Гигантские волны прокатились бы по всему земному шару, с востока на запад, и каждый берег, обращенный на восток, пережил бы самый грандиозный шторм в своей истории. Сначала сушу накрыло бы ослепляющее облако водяной пыли, а за ним пришла бы грохочущая стена воды наподобие цунами. В некоторых местах эти волны проникли бы на много километров вглубь побережья.

Ураганы выбросили бы в атмосферу огромное количество пыли и обломков. Одновременно с этим плотное облако тумана сформировалось бы над холодными поверхностями океана. В обычной ситуации это привело бы резкому падению температур. Так бы и произошло. Во всяком случае, на одной стороне Земли.

Если бы Земля перестала вращаться, нормальный цикл дня и ночи прекратил бы свое существование. Солнце продолжало бы свое видимое движение по небу, но восходы и заходы случались бы теперь раз в год. День и ночь длились бы по шесть месяцев, даже на экваторе. На дневной половине Земли поверхность постоянно жарилась бы на солнце, тогда как на ночной стороне температура резко бы снизилась. Конвекция (перемешивание теплых и холодных слоев воздуха) на дневной стороне привела бы к возникновению мощных штормов и ураганов.

Земля стала бы отчасти напоминать Венеру в начале существования Солнечной системы. Специфика вращения Венеры такова, что, как и наша остановившаяся Земля, эта планета месяцами обращена к Солнцу одной и той же стороной. Однако в ее плотной атмосфере весьма быстрая циркуляция, благодаря чему температура на дневной и ночной стороне приблизительно одинакова.

Хотя длина дня изменилась бы, продолжительность месяца осталась бы прежней. Луна не прекратила бы вращаться вокруг Земли, однако перестала бы удаляться от Земли (как она это делает сейчас из-за приливных сил) и медленно начала приближаться к нам снова.

Более того, Луна сможет в какой-то мере исправить разрушения. Сейчас Земля вращается вокруг своей оси быстрее, чем Луна вокруг Земли, и притяжение нашего спутника замедляет вращение Земли, одновременно отталкивая от нас Луну[2 - См. мой комикс http://what-if.xkcd.com/26 (http://what-if.xkcd.com/26), в котором объясняется, почему это происходит.]. Если бы Земля перестала вращаться, Луна перестала бы удаляться от нас и вместо того, чтобы замедлять Землю, начала бы ее ускорять. Потихоньку, помаленьку гравитация Луны тянула бы нашу планету за собой…

И Земля снова начала бы вращаться.

Релятивистский бейсбольный мяч

ВОПРОС: А что, если попытаться отбить бейсбольный мяч, брошенный со скоростью в 90 % от скорости света?

    – Эллен Макмэнис

Оставим в стороне вопрос о том, как мы заставили мяч лететь так быстро. Предположим, что это был обычный бросок, просто в тот момент, когда подающий игрок (пичтер) бросил мяч, тот загадочным образом ускорился до 0,9 с. Дальше все происходит по обычным законам физики.

ОТВЕТ: Похоже, ответ будет таким: случится очень многое, случится очень быстро, и ничего хорошего ни для отбивающего игрока (баттера), ни для питчера из этого не выйдет.

Мяч будет лететь так быстро, что мир вокруг него станет практически неподвижным. Даже молекулы воздуха фактически замрут. Они будут вибрировать со скоростью несколько сотен км/ч, но мяч будет лететь сквозь них со скоростью чуть меньше 1 млрд км/ч, так что по сравнению с мячом молекулы можно считать неподвижно подвешенными.

Принципы аэродинамики здесь неприменимы. В обычных условиях воздух обтекает предметы, летящие через него, но у молекул воздуха, которые окажутся перед нашим мячом, просто не будет времени отлететь в сторону. Мяч врежется в них с такой силой, что между атомами молекул воздуха и атомами поверхности мяча начнется настоящая реакция ядерного синтеза. Каждое столкновение молекул будет приводить к выбросу гамма-излучения и рассеянных при столкновении частиц[3 - После того, как я впервые опубликовал все эти расчеты, физик из Массачусетского технологического института по имени Ханс Риндеркнехт рассказал мне, что он смоделировал этот сценарий на компьютерах своей лаборатории. Выяснилось, что в начале полета мяча большая часть молекул воздуха двигалась слишком быстро, чтобы это привело к синтезу: они просто пролетали бы сквозь мяч, нагревая его более медленно и равномерно, чем описано у меня.].

Эти гамма-лучи и частицы будут разлетаться, образуя раздувающийся пузырь, центр которого будет находиться в точке, где стоял питчер. Они начнут разрушать молекулы воздуха, выбивая из ядер электроны и превращая воздух на стадионе в расширяющуюся сферу раскаленной плазмы. Стенка этого раздувающегося пузыря будет со скоростью света приближаться к баттеру, лишь немного обгоняя сам мяч.

Постоянный ядерный синтез, происходящий перед мячом, будет оказывать
Страница 3 из 15

на него давление, замедляя его, как если бы мяч был ракетой, которая летит хвостом вперед, включив двигатели. К сожалению, мяч будет двигаться так быстро, что даже невероятная сила термоядерного взрыва замедлит его совсем чуть-чуть. Однако эта сила начнет постепенно уничтожать поверхность мяча, и мельчайшие ее фрагменты будут разлетаться во все стороны. Они полетят так быстро, что при столкновении с молекулами воздуха запустят еще два-три раунда ядерного синтеза.

Спустя примерно 70 наносекунд мяч прилетит к базе. Баттер не успеет даже увидеть, как питчер бросил его, так как свет, несущий эту информацию, достигнет баттера примерно в тот же момент, что и мяч. Столкновения молекул в воздухе к этому моменту практически полностью уничтожат мяч, и он будет представлять собой несущееся как пуля облако расширяющейся плазмы (в основном состоящей из углерода, кислорода, водорода и азота), которое будет все так же врезаться в молекулы воздуха и запускать все больше реакций синтеза. Сначала до баттера доберется оболочка пузыря, состоящая из рентгеновских лучей, а спустя несколько наносекунд на него обрушится облако из осколков мяча.

Центр этого облака к тому моменту, когда оно достигнет базы, все еще будет перемещаться со скоростью, составляющей значительную часть скорости света. Когда центр облака столкнется с битой баттера, то и он, и база будут отброшены волной и пробьют ограждение поля, одновременно расщепляясь на молекулы. Оболочка из гамма-лучей и раскаленной плазмы будет расширяться в стороны и вверх, поглотит бейсбольное поле, обе команды, зрителей, окружающие кварталы, и все это в первую же микросекунду.

Представьте, что вы наблюдаете за этим с холма, расположенного вне города. Первое, что вы увидите, – ослепляющий свет, гораздо более яркий, чем солнце. Он постепенно тускнеет в течение нескольких секунд, и растущий огненный шар превращается в грибовидное облако. Затем раздается нарастающий грохот и приходит взрывная волна, ломающая деревья и сравнивающая с землей дома.

Все в радиусе примерно полутора километров от парка будет снесено до основания, и огненная буря поглотит окружающий город. Бейсбольная площадка превратится в довольно приличного размера кратер, центр которого будет в сотне метров за тем местом, где еще недавно стоял баттер.

Правило Бейсбольной лиги 6.08 (b) гласит, что в данной ситуации баттер явно «получил удар при подаче», а значит, может продвинуться на первую базу.

Купание в ядерном бассейне

ВОПРОС: А что, если искупаться в бассейне для отработавшего ядерного топлива?

    – Джонатан Бастьен-Фильятро

ОТВЕТ: Если вы хорошо плаваете, то вы, вероятно, сможете продержаться на поверхности 10–40 часов. После этого вы потеряете сознание от изнеможения и утонете. То же самое, кстати, верно и для обычного бассейна, на дне которого не хранится ядерное топливо.

Отработавшее ядерное топливо из реактора крайне радиоактивно. Но вода очень хорошо изолирует и охлаждает топливо, и поэтому его можно хранить на дне в течение нескольких десятилетий, пока его активность не снизится до такой степени, что топливо можно будет переместить в сухие контейнеры. Человечество пока не придумало, куда девать эти контейнеры потом, и в течение ближайших десятилетий нам предстоит решить эту проблему.

Вот разрез типичного бассейна для отработавшего топлива:

Температура не будет серьезной проблемой. Теоретически вода в подобном бассейне может разогреться до 50°С, но на практике температура обычно составляет 25–35°С. Это теплее, чем в обычном бассейне, но прохладнее, чем горячая ванна.

Наиболее радиоактивны стержни, лишь недавно извлеченные из реактора. В случае с видами радиации, излучаемыми отработавшим ядерным топливом, каждые 7 см воды уменьшают степень излучения вдвое. Согласно данным компаний, управляющих атомными электростанциями, зоны поражения для «свежих» стержней будут такими:

Если нырнуть на самое дно, коснуться «свежего» стержня локтем и немедленно вынырнуть на поверхность, то этого, вероятно, будет достаточно для того, чтобы умереть от излучения.

Однако за пределами опасной зоны можно плавать сколько угодно – доза излучения, которую вы получите, будет меньше, чем обычное фоновое излучение в повседневной жизни. Вода защищает вас от большей части этой фоновой дозы, так что, плавая в бассейне для отработавшего ядерного топлива, вы получите меньшую дозу радиации, чем просто прогуливаясь по улице.

Но все это верно лишь при условии, что с самим бассейном все в порядке. Если же стенка контейнера, содержащего стержень, будет повреждена коррозией, то в воде могут оказаться продукты деления ядра. Воду в подобных бассейнах довольно эффективно очищают, и плавать в ней не опасно, но при этом она достаточно радиоактивна, чтобы ее нельзя было разливать в бутылки и продавать[4 - А жаль, получился бы отличный энергетический напиток.].

Напоминаю:я просто рисую комиксы! Если вы, стоя на краю радиоактивного бассейна, принимаете мои слова за советы по безопасности, то вы, вероятно, заслуживаете всего того, что с вами вот-вот случится.

Мы точно знаем, что в бассейнах для отработавшего топлива можно плавать, потому что их регулярно обслуживают ныряльщики. Однако этим ныряльщикам нужно соблюдать осторожность.

21 августа 2010 года ныряльщик работал в бассейне Лейбштадского ядерного реактора в Швейцарии. Он увидел на дне бассейна кусок какого-то шланга и связался с диспетчером, чтобы узнать, что делать. Ему велели поднять шланг и положить его в сумку с инструментами, что ныряльщик и сделал. Из-за бульканья воды в бассейне он не услышал, как сработал его датчик радиации.

Когда сумку с инструментами вытащили из воды, заработали все датчики радиации в здании. Сумку тут же бросили обратно в воду, а ныряльщик быстро вылез из бассейна. Дозиметры показали, что он получил высокую дозу радиации, причем особенно высоким было облучение его правой руки.

«Шланг» оказался фрагментом защитной оболочки радиационного монитора в ядре реактора, крайне радиоактивным за счет нейтронного потока. Этот кусок случайно отрезали от обшивки в 2006 году, когда закрывали контейнер, и с тех пор он валялся на дне в дальнем углу бассейна, где его никто не замечал в течение четырех лет.

Этот предмет был настолько радиоактивен, что если бы ныряльщик засунул его за пояс или в положил в рюкзак, где «шланг» оказался бы близко к телу, доза оказалась бы смертельной. К счастью, в данной ситуации рабочего защитила вода, и только рука – часть тела, куда более успешно противостоящая радиации, чем уязвимые внутренние органы, – получила серьезную дозу облучения.

Короче говоря, с вами, скорее всего, все будет в порядке, если вы не будете нырять на дно и подбирать там что попало.

Но чтобы быть окончательно в этом уверенным, я связался с моим приятелем, который работает на исследовательском реакторе, и спросил его: что, по его мнению, произойдет с человеком, который захочет поплавать в их бассейне для хранения топлива?

«В нашем бассейне? – он на секунду
Страница 4 из 15

задумался. – Умрет еще до того, как доберется до воды: его пристрелит охрана».

Странные (и тревожные) вопросы из папки «Входящие» сайта «А что, если?»

ВОПРОС: Можно ли охладить зубы настолько, чтобы они треснули при попытке выпить горячий кофе?

    – Шелби Хеберт

ВОПРОС: Сколько домов сгорает в США каждый год? И как проще всего значительно увеличить эту цифру (хотя бы на 15 %)?

    – Аноним

Машина времени по-нью-йоркски

ВОПРОС: Когда путешествуешь во времени, всегда оказываешься в одной и той же точке пространства. Во всяком случае, в фильме «Назад в будущее» дело обстоит именно так. А что, если, стоя на Таймс-сквер, отправиться на 1000 лет назад? А на 100 000? А на миллиард лет? А что, если отправиться на миллион лет вперед?

    – Марк Деттлинг

Тысячу лет назад

Манхэттен постоянно населен последние 2000 лет, а первые люди появились здесь приблизительно 9000 лет назад.

В XVII веке, когда прибыли европейцы, в этих краях жили индейцы ленапе[5 - Также известные как делавары или ленни-ленапе.].

Тысячу лет назад это место, вероятно, тоже населяла некая группа племен, но они жили за пять сотен лет до контакта с европейцами и были так же далеки от ленапе XVII века, как индеец XVII века от современного американца.

Чтобы увидеть, как выглядела Таймс-сквер до того, как появился город, стоит обратиться к замечательному проекту под названием Welikia, выросшему из маленького проекта Mannahatta. Разработчики создали подробную экологическую карту нью-йоркского ландшафта времен прибытия первых европейцев.

Интерактивная карта, доступная на сайте welikia.org, представляет собой воображаемый спутниковый снимок «другого» Нью-Йорка. В 1609 году остров Манхэттен был покрыт холмами, болотами и рощами, озерами и реками.

Район Таймс-сквер тысячелетней давности выглядел, вероятно, примерно так, как это показывает Welikia. Если не вдаваться в подробности, местный пейзаж, скорее всего, напоминал девственные леса, которые все еще можно увидеть на северо-востоке США. Однако были бы и серьезные отличия.

Тысячу лет назад на Манхэттене водились довольно крупные животные. В тех разрозненных остатках девственных лесов, что мы видим сегодня, практически нет крупных хищников, разве что несколько медведей, да немного волков и койотов, а вот пумы практически отсутствуют (зато популяции оленей, напротив, разрослись, благодаря в том числе и исчезновению крупных хищников).

В лесах Манхэттена тысячелетней давности в изобилии росли каштаны. До эпидемии паразитического гриба в начале XX века леса на востоке США на 25 % состояли из каштанов, от которых сегодня остались только пни.

Эти пни все еще можно увидеть в лесах Новой Англии. Периодически они дают новые побеги, которые, однако, быстро увядают под действием гриба. Однажды (и это будет довольно скоро) исчезнут последние из этих пней.

А в лесах тысячелетней давности вам встретятся волки, особенно когда вы будете продвигаться вглубь материка. Еще вы увидите пум[6 - Также известных как горные львы, они же кугуары, они же горные кошки.] и странствующих голубей[7 - Хотя вы вряд ли увидите зрелище, которое представилось европейским поселенцам: миллиарды птиц, закрывающих небо. В своей книге «1491» историк Чарльз Манн предполагает, что эти гигантские стаи могли возникнуть именно в результате колонизации и связанного с ней хаоса в экосистеме, потревоженной появлением оспы, мятлика и медоносных пчел.].

А вот чего вы точно не увидите, так это дождевых червей. Их не было в Новой Англии до появления европейских колонистов. Чтобы узнать, почему, давайте отправимся еще глубже в прошлое.

10 000 лет назад

Земля 10 000 лет назад только-только выбралась из очень холодного периода. Ледники, покрывавшие Новую Англию, уже исчезли. 22 000 лет назад южная граница льда проходила у Статен-Айленда[8 - Остров в устье р. Гудзон, самый южный округ города Нью-Йорк. – Прим. ред.], но уже через четыре тысячи лет назад отступила на север за Йонкерс[9 - Город на Гудзоне к северу от Нью-Йорка. – Прим. ред.]. К моменту нашего прибытия (еще спустя восемь тысяч лет) ледник по большей части отодвинулся к северу за пределы современной канадской границы.

Ледники сгладили ландшафт, обнажив скальные породы. В течение 10 000 лет жизнь медленно возвращалась вслед за ледником обратно на север. Некоторые виды перемещаются быстрее, чем другие, и когда европейцы пришли в Новую Англию, дождевые черви еще не успели вернуться сюда.

Ледники отступал, но гигантские ледяные горы откалывались от него и начинали медленно таять.

На месте этих ледяных гор остались заполненные водой впадины, которые называют ледниковыми озерами. Озеро Окленд-лейк в нью-йоркском округе Квинс одно из таких озер. А еще ледники тащили с собой валуны, и некоторые из этих ледниковых (эрратических) валунов можно и сегодня увидеть в Центральном парке.

Подо льдом, под большим давлением, текли реки талой воды, принося с собой песок и гравий. Эти отложения в виде земляных валов (так называемые эскеры) исчерчивают леса вокруг моего дома в Бостоне. На их счету немало странных образований в ландшафте, включая русла рек, имеющие в сечении форму буквы U.

100 000 лет назад

Пейзаж 100 000 лет назад выглядел, вероятно, во многом так же, как сегодня[10 - Разве что рекламных щитов было поменьше.]. Мы живем в эпоху коротких, быстро сменяющих друг друга похолоданий и потеплений, но в течение последних 10 000 лет наш климат был стабильным[11 - Был. Но уже скоро мы с этим покончим.] и теплым.

Сто тысяч лет назад Земля приближалась к концу аналогичного периода климатической стабильности. Уже закончилось так называемое Эемское межледниковье, и, скорее всего, в этот период флора и фауна региона показались бы нам весьма знакомыми.

А вот топография прибрежной зоны была бы совершенно иной: острова Статен-Айленд, Лонг-Айленд, Нантакет и Мартас-Винъярд в то время были полуостровами, которые окончательно отделит от материка лишь более поздний ледник, который пройдет по этому краю, словно бульдозер. Сотню тысяч лет назад прибрежные воды были испещрены совсем другими островами.

В лесах того времени водились многие современные животные, например птицы, белки, олени, волки, черные медведи, но встречались и удивительные исключения. Чтобы узнать о них, обратимся к тайне вилорога.

Современный вилорог – это загадка природы. Он быстро бегает, гораздо быстрее, чем необходимо. Он разгоняется до 88 км/час и способен несколько минут бежать с этой скоростью, покрыв за это время 5–6 километров. Однако его основные враги, волки и койоты, едва достигают скорости в 56 км/ч. Для чего же вилорог такой быстрый?

Ответ заключается в том, что мир, в котором вилорог эволюционировал, был куда опаснее современного. Сотню тысяч лет назад североамериканские леса были ареалом обитания так называемого волка ужасного (Canis dirus), малого короткомордого медведя (Arctodus pristinus) и смилодона (Smilodon fatalis). Все эти звери были, по всей видимости, быстрее и опаснее современных хищников. Все они исчезли во время плейстоцен-голоценового вымирания, которое произошло вскоре после того, как на континенте появились
Страница 5 из 15

первые люди[12 - Если что – это случайное совпадение.].

Если же мы углубимся еще дальше в прошлое, то встретим еще одного устрашающего хищника.

1 000 000 лет назад

Миллион лет назад, до наступления последнего крупного ледника, в мире было довольно тепло. Прошла половина четвертичного периода, великие современные ледниковые эпохи начались за несколько миллионов лет до того, но наступление и отступление ледников в этот момент замедлилось, а климат был довольно стабильным.

Хищникам, которых мы встретили ранее, быстроногим созданиям, способным охотиться на вилорога, составлял компанию еще один безжалостный убийца – длинноногая гиена, напоминавшая современного волка. Гиены в основном водились в Азии и Африке, но во время снижения уровня моря один вид гиен пересек Берингов пролив и попал в Северную Америку. За это достижение этот вид гиены назвали Chasmaporthetes, что значит «тот, кто видел каньон».

А затем вопрос Марка отправляет нас в еще более далекое прошлое.

1 000 000 000 лет наза д

Миллиард лет назад континентальные плиты были собраны в один огромный суперконтинент. Это была не хорошо известная Пангея, но ее предшественник Родиния. Геологические данные не отличаются особой точностью, но мы предполагаем, что выглядела она примерно вот так:

Во времена Родинии скальные породы, подстилающие Манхэттен, еще не сформировались, но часть континента, из которой впоследствии образовалась Северная Америка, уже была древней. Сегодняшний Манхэттен, вероятно, входил в состав большого куска суши, из других частей которого образовались территории Анголы и Южно-Африканской Республики.

В этом первобытном мире не было ни растений, ни животных. Жизнь наполняла океаны, но это была простая, одноклеточная жизнь: на поверхности воды плавали ковры из сине-зеленых водорослей. Эти скромные существа – самые смертоносные убийцы за всю историю жизни на Земле.

Сине-зеленые водоросли, или цианобактерии, были первыми фотосинтезирующими организмами. Они вдыхали углекислый газ и выдыхали кислород. Кислород весьма активное вещество, он заставляет железо ржаветь (окисление), дерево гореть (активное окисление). Когда цианобактерии только появились, кислород, который они выдыхали, был токсичен почти для всех остальных форм жизни. Вымирание видов, произошедшее в результате появления цианобактерий, называют кислородной катастрофой.

Но после того как цианобактерии наполнили атмосферу Земли и ее воду токсичным кислородом, развились создания, которые воспользовались активной природой этого газа, чтобы запустить новые биологические процессы. Мы наследники тех первых существ, которые научились дышать кислородом.

Многие подробности этой истории остаются неясными – непросто реконструировать мир, каким он был миллиард лет назад. Но сейчас вопрос Марка отправит нас в еще более неясное – время в будущее.

Миллион лет спустя

Когда-нибудь человечество вымрет. Никто не знает, когда именно[13 - Если вы знаете, напишите мне, пожалуйста.], но ничто не живет вечно. Быть может, мы улетим к звездам и проживем миллиарды лет. А может быть, наша цивилизация рухнет, а мы все погибнем от болезней и голода, и последних из нас съедят бездомные кошки. Или нас всех убьют нанороботы, захватившие планету спустя всего несколько десятков лет после того, как вы прочли это предложение. Точно сказать нельзя.

Миллион лет – это много. Это в несколько раз дольше, чем существует Homo sapiens, и более чем в сто раз дольше, чем существует письменность. Разумно предположить, что, как бы ни разворачивалась история человечества, через миллион лет ее нынешняя фаза закончится.

Но геологические силы Земли продолжат свою работу и без нас. Ветра, дожди и песчаные бури уничтожат и похоронят артефакты нашей цивилизации. Изменения климата, вызванные человеческой деятельностью, возможно, отсрочат начало нового ледникового периода, но сам цикл этих периодов мы не прервали, и однажды ледники придут снова. Мало что останется от человечества спустя миллион лет.

Скорее всего, самой живучей реликвией нашего времени будет слой пластика, которым мы покрыли всю планету. Добывая нефть, превращая ее в прочные, надолго сохраняющиеся полимеры и разбрасывая их по поверхности Земли, мы оставили след, который может пережить все прочие наши достижения.

Пластик раскрошится, будет захоронен в почве, и, возможно, какие-нибудь микробы научатся его перерабатывать, но весьма вероятно, что и спустя миллион лет толстый слой обработанных углеводородов – фрагменты наших бутылок из-под шампуня и пластиковых пакетов – будет служить химическим памятником цивилизации.

Еще более отдаленное будущее

Солнце становится все ярче. В течение трех миллиардов лет сложная система обратных связей поддерживала температуру Земли относительно стабильной, пока Солнце понемногу теплело.

Но через миллиард лет эта система даст сбой. Наши океаны, которые вскармливали и охлаждали жизнь, превратятся в ее злейшего врага. Они закипят на горячем солнце, окружат планету толстым слоем водяного пара и усилят парниковый эффект. Через миллиард лет Земля станет второй Венерой.

Нагреваясь еще больше, планета может полностью лишиться воды, и ее атмосфера наполнится каменным паром, поскольку начнет кипеть и испаряться и поверхность суши. Спустя еще несколько миллиардов лет растущее Солнце в конце концов поглотит Землю.

Земля исчезнет, и множество молекул, из которых состояла площадь Таймс-сквер, разлетятся прочь от умирающего Солнца. Эти пылевые облака будут плыть через космос, возможно, рождая по пути новые звезды или планеты.

Если люди к тому времени выберутся за пределы Солнечной системы и переживут Солнце, не исключено, что наши потомки будут жить на одной из этих планет. Атомы Таймс-сквер, пройдя сквозь горнило Солнца, сформируют наши новые тела.

И в один прекрасный день мы все либо вымрем, либо станем ньюйоркцами до мозга костей.

Где моя вторая половинка?

ВОПРОС: А что, если бы у каждого человека на самом деле была лишь одна-единственная «вторая половинка» – некий человек, живущий неизвестно где?

    – Бенджамин Стаффин

ОТВЕТ: Это был бы сущий кошмар.

С концепцией единственной второй половинки вообще много проблем. Как поет австралийский комик Тим Минчин,

Твоя любовь одна на миллион,

Ее не купишь ни за какую цену,

Но статистика говорит,

Что из оставшихся 999 000

Некоторые точно будут не хуже.

Так что, если бы у нас действительно имелась одна-единственная вторая половинка и больше мы ни с кем в мире не были бы счастливы? Удалось бы нам встретить свою половинку?

Предположим, что наша половинка предопределена при рождении. Вы ничего не знаете о том, кто этот человек и где он живет, но любовные романы учат нас, что вы узнаете друг друга, как только встретитесь взглядом.

Сразу возникают некоторые вопросы. Во-первых, жива ли еще ваша вторая половинка? За всю историю человечества на Земле жило около сотни миллиардов людей, но сегодня нас только семь миллиардов (таким образом, человеческая жизнь до сих пор приводила к смерти в 93 % случаев). Иными словами 93 % вторых
Страница 6 из 15

половинок уже нет в живых.

Это ужасно! Но погодите, дальше будет только хуже. Простая логика подсказывает, что нельзя ограничиваться только людьми, жившими в прошлом: надо учитывать и неизвестное число людей будущего. Если ваша вторая половинка может жить в прошлом, значит, некоторые половинки могут находиться и в будущем, ведь и вы сами такая «будущая» половинка для кого-то в прошлом.

Теперь давайте предположим, что ваша половинка живет в то же время, что и вы. Предположим также, что вы сверстники в пределах нескольких лет (это более строгое ограничение, чем известное правило «половина вашего возраста плюс семь лет»[14 - См. http://xkcd.com/314 (http://xkcd.com/314).], однако это правило работает, только если половинки встречаются уже взрослыми – скажем, одному 30, а другому 40 лет, и не работает, если бы они встретились лет на 15 лет раньше). С учетом этих ограничений у каждого из нас окажется примерно полмиллиарда потенциальных половинок.

Но как насчет пола и ориентации? Культуры? Языка? Мы можем и дальше использовать демографические категории, чтобы еще более сузить круг потенциальных партнеров, однако при этом мы уходим от идеи случайной половинки. В нашем сценарии вы с вашей половинкой ничего не знаете друг о друге, пока не посмотрите друг другу в глаза. Можно сказать, что ваша ориентация будет определяться исключительно вашей половинкой.

Как видим, шансы столкнуться со своей половинкой весьма малы. Количество незнакомцев, которым мы смотрим в глаза каждый день, варьируется от нуля (если вы отшельник или житель маленького городка) до нескольких тысяч (если вы полицейский на Таймс-сквер), но давайте предположим, что вы встречаетесь взглядом с несколькими десятками незнакомцев в день (я скорее интроверт, поэтому для меня это довольно щедрое предположение). Если 10 % из них примерно ваши сверстники, то всего за всю жизнь вы поймаете взгляд примерно 50 000 людей. С учетом того, что у каждого из нас есть 500 000 000 потенциальных половинок, истинную любовь в течение своей жизни обретет лишь один из 10 000…

Если опасность умереть в одиночестве столь велика, то общество просто обязано придумать способ увеличить вероятность визуального контакта. Например, построить огромные транспортные ленты, чтобы граждане могли перемещаться, глядя в глаза визави… А если того же эффекта можно достичь с помощью веб-камеры, то надо разработать модифицированную версию сайтов для случайных знакомств – таких, например, как Chat Roulette.com.

Если бы каждый из нас использовал эту систему по восемь часов в день семь дней в неделю и если бы нам требовалась бы всего пара секунд, чтобы определить, является ли человек нашей половинкой, то система могла бы теоретически свести всех людей со своими половинками за несколько десятилетий. (Я произвел несколько несложных вычислений, чтобы в целом оценить шансы участников. Если вы хотите рассчитать ваш конкретный случай с помощью математических инструментов, можете начать с задач на перестановку.)

Но в реальной жизни многим людям и без того непросто найти хоть какое-то время для личной жизни, и мало кто смог бы посвятить ее устройству два десятилетия. Так что не исключено, что только детки богатых родителей смогли бы часами сидеть на нашем сайте (назовем его «рулетка-для-половинок-точка-ком»). Но, увы, к несчастью для легендарного «золотого процента» (считается, что богачи составляют 1 % мирового населения), большая часть их половинок входила бы в оставшиеся 99 %… И если лишь один процент от «золотого процента» воспользуется нашей рулеткой, то лишь один процент от процента от процента найдет свою пару – то есть один из 10 000 человек.

Зато у оставшихся 9 999 богачей появился бы стимул втянуть в эту систему больше людей. В результате могли бы возникнуть благотворительные проекты, направленные, например, на то, чтобы у каждого жителя Земли оказался компьютер – нечто среднее между кампанией «Ноутбук каждому ребенку» и усовершенствованным сайтом знакомств. Профессии кассира в супермаркете или патрульного на Таймс-сквер стали бы невероятно престижными, поскольку дают возможность часто встречаться глазами с другими. Люди рвались бы в большие города и на разнообразные тусовки, чтобы найти свою любовь, точно так же, как они это делают и сейчас.

Но даже если бы кто-то из нас провел годы на сайте «рулетка-для-половинок-точка-ком», кто-то нашел работу, где постоянно заглядывал в глаза незнакомцам, а кто-то просто надеялся на удачу, – лишь малая часть всех нас нашла бы свою любовь. Остальным бы не повезло.

В условиях постоянного стресса и давления новой общественной нормы кто-то начал бы притворяться. Всем хочется вступить в клуб счастливых, и двое одиночек могли бы объединиться, чтобы разыграть мнимую встречу двух половинок. Поженившись, они продолжали бы скрывать свою выдумку и старались изобразить счастливейшую пару на глазах у своих друзей и своей семьи.

Мир случайных половинок был бы очень одиноким местом. Давайте надеяться, что наш мир не таков.

Лазерная указка

ВОПРОС: А что, если все люди на Земле одновременно направят на Луну лазерные указки? Изменит ли она свой цвет?

    – Питер Липовиц

ОТВЕТ: Нет, если речь идет об обычной указке.

Во-первых, следует учесть, что не все мы видим Луну одновременно. Поскольку примерно 75 % населения Земли живет между нулевым и 120-м меридианом, идеальное положение Луны для нашего эксперимента где-то над Аравийским морем.

Какую луну выбрать – новую или полную? С одной стороны, луна в новолуние гораздо темнее, и свет наших лазеров было бы проще заметить. Но с другой стороны, такая луна – сложная мишень, поскольку видно ее в основном днем, а значит, результат наших усилий практически не будет виден.

Давайте лучше возьмем фазу четверти луны, чтобы можно было сравнивать эффект наших лазеров на темной и светлой стороне.

Это наша мишень

Обычная красная лазерная указка имеет мощность примерно в 5 милливатт, так что ее свет вполне сможет достичь Луны, хотя и рассеется по достаточно большому участку лунной поверхности. Земная атмосфера немного исказит луч и частично поглотит его, но большая часть света все же достигнет цели.

Давайте предположим, что все мы достаточно меткие, чтобы вообще попасть в Луну, но не все попадем в одно и то же место, так свет распределится по поверхности равномерно.

В 00:30 по Гринвичу все прицеливаются и нажимают кнопку!

Вот что произойдет.

Что ж, довольно обидно… Но такого результата и следовало ожидать. Солнце освещает Луну с мощностью, превышающей киловатт на квадратный метр. Поскольку площадь экваториального сечения Луны составляет около 10?? м?, ее омывает примерно 10

ватт солнечного света то есть 10 петаватт, или 2 мегаватта на каждого жителя Земли, что намного превышает мощность наших 5-милливаттных лазерных указок. В каждой части этих расчетов есть некоторые неточности, но в целом соотношение именно таково.

Лазер мощностью в 1 Вт – очень опасная вещь. Он не просто способен ослепить вас, он может обжечь кожу и даже поджечь окружающие предметы.
Страница 7 из 15

Совершенно логично, что его нет в свободной продаже в США… Шучу-шучу! Есть, конечно, и стоит он всего 300 долларов.

Итак, предположим, мы потратили два миллиарда долларов, чтобы купить каждому жителю Земли зеленый лазер мощностью 1 Вт. (Примечание для кандидатов в президенты – подобный пункт в Вашей программе помог бы Вам получить мой голос.) Этот лазер не просто более мощный, чем лазерная указка, – зеленый цвет находится ближе к середине видимого спектра, поэтому глаза воспринимают его лучше, и он кажется более ярким.

Вот какой будет эффект.

Упс!.. Наши лазерные указки посылают 150 люменов света (больше, чем фонарики), и ширина их луча составляет 5 угловых минут. Такой луч осветит поверхность Луны лишь примерно на ? люкса (лк) по сравнению с освещенностью в 130 000 лк, которую обеспечивает нашему спутнику Солнце. Даже если все мы прицелимся с идеальной точностью, это даст нам дополнительно всего лишь около 5 лк на примерно 10 % поверхности Луны.

Для сравнения – полная Луна освещает поверхность Земли примерно на 1 лк. Это значит, что действия наших лазеров не видно не только с Земли: даже стоящему на Луне астронавту отсвет земных указок на лунной поверхности показался бы более слабым, чем лунный свет на Земле.

В последнее десятилетие с развитием литиевых батарей и светодиодов появилось очень много самых разнообразных фонарей, но очевидно, что карманный фонарик нам в любом случае не поможет. Так что пропустим все это и выдадим каждому участнику проекта устройство Nightsun.

Название может показаться вам незнакомым, но есть шанс, что вы видели это устройство в действии: так называются прожекторы, которые установлены на вертолетах полиции и береговой охраны. Испуская 50 000 люменов света, они вполне способны превратить ночь в день.

Луч такого устройства имеет угловую ширину несколько градусов, так что нам понадобятся фокусирующие линзы, чтобы сузить его до половины градуса, необходимой, чтобы попасть в Луну.

Вот полученный эффект.

Его почти не видно, но все же прогресс есть! Луч лазера дает освещенность в 20 лк, то есть он ярче, чем темная половина лунного диска. Однако разглядеть это очень сложно, и мы точно не увидим никаких следов луча на светлой половине Луны.

Давайте заменим каждый Nightsun на проектор IMAX 30 000-ваттную пару ламп с системой водяного охлаждения, которые вместе выдают более миллиона люменов.

И все равно едва заметно… На крыше отеля «Люксор» в Лас-Вегасе стоит самый мощный проектор на Земле. Давайте выдадим такой же каждому участнику нашего эксперимента.

Ах да, и добавим линзы, чтобы весь луч фокусировался именно на Луне.

Наш свет теперь точно виден, так что мы достигли своей цели. Поздравляю команду.

Кхм… Вообще-то министерство обороны США в свое время проектировало мегаваттные лазеры для уничтожения боеголовок в полете. Таким был Boeing YAL-1–химический кислородно-йодный мегаваттный лазер, который устанавливали на самолете Boeing-747. Это инфракрасный лазер, так что мы не увидим его луча, но давайте вообразим, что существует лазер аналогичной мощности, излучающий свет в рамках видимого спектра.

Наконец-то нам удалось сравняться с яркостью солнечного света! Правда, мы потратили на это пять петаватт энергии, что вдвое больше, чем в среднем тратит человечество за то же самое время.

Ладно, давайте установим по мегаваттному лазеру на каждом квадратном метре территории Азии. Правда, ради поддержания работы 50 миллиардов лазеров придется израсходовать все запасы нефти на Земле примерно за две минуты, зато в течение этих двух минут Луна будет выглядеть так:

Луна будет казаться такой же яркой, как полуденное солнце, и к концу этих двух минут лунная поверхность разогреется настолько, что начнет светиться сама.

Что ж, сделаем еще один уверенный шаг за пределы правдоподобия. Самый мощный лазер на земле находится в National Ignition Facility – лаборатории по исследованию управляемой термоядерной реакции. Это ультрафиолетовый лазер с мощностью 500 тераватт. Он испускает отдельные импульсы продолжительностью по несколько наносекунд, так что энергия одного импульса будет равна энергии, которую можно получить при сжигании примерно четверти стакана бензина.

Представим, что мы каким-то образом нашли способ заставить этот лазер постоянно работать, выдали каждому жителю Земли по экземпляру устройства и одновременно направили бы все эти устройства на Луну.

К сожалению, такой поток энергии обратит атмосферу на пути луча в плазму, которая немедленно подожжет поверхность Земли и убьет нас всех. Но давайте предположим, что этот лазер как-то умудряется проходить через атмосферу Земли, не взаимодействуя с ней.

Но даже при этих условиях, как выясняется, Земля все равно загорится. Свет, отраженный от Луны, будет в 4000 раз ярче света полуденного солнца. В таком сиянии океаны Земли выкипят меньше чем за год.

Но забудем на минуту о Земле. А что же будет с Луной?

Давление света ускорит вращение Луны примерно на одну десятимиллионную часть ее гравитационной силы. Это ускорение будет не слишком заметно в краткосрочной перспективе, но со временем его хватит, чтобы Луна переместилась на более высокую орбиту вокруг Земли…

…Но если бы давление света было бы единственным фактором, который повлияет на Луну!

Сорока мегаджоулей энергии достаточно, чтобы испарить килограмм скальной породы. Если предположить, что лунная порода имеет среднюю плотность 3 кг/л, то лазеры дадут достаточно энергии, чтобы поверхность нашего спутника начала испаряться и на ней образовался бы кратер, глубина которого будет увеличиваться со скоростью четыре метра в секунду:

Однако в реальности лунная порода будет испаряться не так быстро по очень важной причине: испарившийся камень не исчезает в небытие. Поверхность Луны превратится в плазму, и эта плазма перекроет путь лучу. Наш лазер будет вливать все больше энергии в плазму, нагревая ее все больше и больше. Частицы плазмы будут сталкиваться, отскакивать друг от друга, врезаться в поверхность Луны и в конце концов с потрясающей скоростью вылетят в космос.

Этот поток вещества по сути превратит всю поверхность Луны в ракетный двигатель, причем на удивление эффективный. Использование лазеров с целью удаления материала с поверхности называется лазерной абляцией, и этот метод весьма перспективен с точки зрения перемещения космических кораблей.

Луна огромна, но каменная плазма начнет медленно и неуклонно отталкивать ее от Земли (этот выброс также отдраит дочиста поверхность Земли и уничтожит, в том числе, и все лазеры, но мы пока притворяемся, что нам ничего не грозит). Одновременно плазма будет уничтожать и лунную поверхность, и это сложное взаимодействие нелегко смоделировать.

Но если предположить, что частицы плазмы отлетают от Луны со скоростью 500 км/с, потребуется несколько месяцев, чтобы Луна вышла за пределы действия наших лазеров. Наш спутник сохранит большую часть своей массы, но выйдет за пределы гравитации Земли и перейдет на асимметричную орбиту вокруг
Страница 8 из 15

Солнца.

Технически Луна не станет новой планетой, если брать определение Международного астрономического союза. Поскольку ее новая орбита будет пересекаться с земной, она будет считаться карликовой планетой, как Плутон. Пересечение орбит Луны и Земли вызовет периодические непредсказуемые колебания. В конце концов Луна может упасть на Солнце, вылететь за пределы Солнечной системы или врезаться в одну из планет – весьма вероятно, нашу.

Вот что я называю настоящей мощностью!

Периодическая стена элементов

ВОПРОС: А что, если построить Периодическую таблицу Менделеева из кубиков, сделанных из соответствующих элементов?

    – Энди Коннолли

ОТВЕТ: Есть люди, которые коллекционируют химические элементы. Они пытаются собрать как можно больше физических образцов и раскладывают их в ящички, составленные в виде таблицы Менделеева.

Из 118 элементов таблицы три десятка – такие как гелий, углерод, алюминий, железо или серу – можно купить в чистом виде в магазинах. Еще несколько десятков можно раздобыть, разломав какой-нибудь прибор (например, образец америция можно найти в детекторе дыма). Еще что-то можно заказать в Интернете.

В общем и целом вам, возможно, удастся собрать примерно 80 элементов или 90, если вы готовы немножко рискнуть своим здоровьем, безопасностью и репутацией законопослушного гражданина. Остальные слишком радиоактивны или недолговечны, чтобы можно было собрать в одном месте за один раз больше нескольких атомов.

А что, если вам это все же удалось бы?

В Периодической таблице Менделеева 7 рядов[15 - К тому моменту, как вы это читаете, могли добавить и восьмой ряд. А если вы читаете это в 2038 году, то в таблице уже наверняка 10 рядов, но любое ее обсуждение запрещено роботами, захватившими нашу планету!].

Верхние два ряда составить легко.

Третий мог бы сильно обжечь вас.

Четвертый убил бы токсичным дымом.

Пятый сделал бы все то же самое плюс облучил бы вас небольшой дозой радиации.

Шестой с грохотом взорвался бы, превратив все вокруг в облако радиоактивного и ядовитого огня и пыли.

А вот строить седьмой ряд я бы вообще не рекомендовал.

Начнем сверху. Первый ряд простой, хотя и скучный.

Кубик водорода поднялся бы кверху и растаял, как воздушный шарик без оболочки. То же случилось бы с гелием.

Второй ряд уже сложнее.

Кубик лития немедленно потемнел бы на воздухе. Бериллий довольно токсичен, так что с ним надо обращаться осторожно и стараться, чтобы его пыль не попала в воздух.

Кубики кислорода и азота будут медленно таять в воздухе, постепенно исчезая[16 - При условии, что все они находятся в двухатомной форме, т. е. О

, N

. Если кубик состоит из отдельных атомов, они немедленно объединятся, разогревшись при этом до нескольких тысяч градусов.]. Неон также уплывет прочь.

Бледно-желтый кубик фтора немедленно стек бы на пол. Надо учесть, что фтор – самый активный окислитель во всей таблице. Почти любое вещество немедленно загорится, соприкоснувшись с ним.

Я спросил специалиста по органической химии Дерека Лоу[17 - Дерек ведет прекрасный химический блог In the Pipeline.], что он думает по этому поводу. Дерек сказал, что фтор не будет реагировать с неоном и у него установится что-то вроде вооруженного перемирия с хлором, но все остальное?.. Пуфф! Фтор будет создавать проблемы и встречаясь с элементами из нижних рядов в таблице, а вступая в контакт с любой жидкостью, образовывал бы чрезвычайно едкую фторную кислоту.

Если вы вдохнете даже следовое количество фтора, это сильно повредит или полностью уничтожит ваш нос, ваши легкие, рот, глаза и, в конечном итоге, все ваше тело. При работе с фтором вам точно потребовался бы противогаз. Но помните, что фтор разъедает многие материалы, из которых делают противогазы, так что последний лучше сначала протестировать. В общем, удачи и вперед, к третьему ряду!

Больше всего неприятностей в третьем ряду следует ожидать от фосфора. Чистый фосфор может иметь несколько форм. Красный фосфор более или менее безопасен. Белый вспыхивает при контакте с воздухом и горит жарким пламенем, которое трудно погасить. Вдобавок ко всему белый фосфор довольно ядовит[18 - Именно из-за этих его свойств белый фосфор используют в печально известных зажигательных бомбах и снарядах.].

Сера в обычных обстоятельствах не представляет проблемы, разве что неприятно пахнет. Однако тут у нас сера зажата между горящим фосфором (слева) и фтором с хлором (справа). При контакте с газообразным фтором сера, как и многие вещества, начинает гореть.

Инертный аргон тяжелее воздуха, так что он просто растекся бы по земле. Но не будем волноваться по поводу аргона, у нас сейчас есть проблемы посерьезнее.

Горение приведет к появлению разнообразных кошмарных химических соединений с названиями наподобие гексафторид серы. Если вы строите нашу стену в замкнутом помещении, то, скорее всего, уже задохнулись от ядовитого дыма, а ваш дом, возможно, сгорел до основания.

И это всего лишь третий ряд. Так вперед же, к четвертому!

Мышьяк – это звучит страшно. И страх этот вполне обоснован. Мышьяк токсичен практически для всех сложных форм жизни. Иногда подобная паника по поводу химических веществ со страшными названиями не обоснована: в нашей еде и воде присутствуют следовые количества мышьяка, и мы с ними отлично справляемся. Но сейчас не тот случай.

Горящий фосфор (к которому теперь присоединился горящий калий, который также склонен к спонтанному самовозгоранию) может поджечь мышьяк, высвободив большое количество триоксида мышьяка. Это довольно ядовитая штука. Не советую ее вдыхать.

Весь этот ряд тоже неважно пахнет. Селен и бром будут яростно вступать в реакции, и Лоу сказал мне, что по сравнению с запахом горящего селена запах серы – «это как духи от Шанель».

Если алюминий переживет этот пожар, с ним произойдет странная вещь. Плавящийся на один ряд ниже галлий потечет на алюминий, нарушив его структуру и сделав его непрочным и мягким, как мокрая бумага[19 - Поищите на Youtube по словам gallium infiltration, чтобы увидеть, насколько это странное зрелище.].

Горящая сера прольется на бром. При комнатной температуре этот элемент представляет собой жидкость, и это его свойство разделяет только еще одно простое вещество – ртуть. И то, и другое – довольно противные штуки. Разброс токсических веществ, возникших к этому моменту в результате горения, уже неисчислимо велик. Однако если вы наблюдаете опыт с безопасного расстояния, у вас есть шансы выжить.

В пятом ряду есть кое-что интересное – технеций-99, наш первый радиоактивный кирпичик.

Технеций – самый легкий элемент из тех, что не имеют стабильных изотопов, и он практически не встречается в природе. Его название и говорит о том, что он был получен искусственно. Доза радиации, которую излучает куб из технеция объемом в один литр, не будет смертельной, если просто вставить его в нашу периодическую стену, но все же она весьма значительна. Если вы проведете весь день, надев на голову полый куб из технеция или вдыхая технециевую пыль, этот элемент вполне может вас
Страница 9 из 15

убить.

Если не считать технеция, пятый ряд будет во многом похож на четвертый.

Вперед, к шестому ряду! Как бы осторожны вы ни были до сих пор, шестой ряд точно вас убьет.

Этот вариант Периодической таблицы несколько больше, чем тот, к которому вы, вероятно, привыкли, так как мы добавили лантаноиды и актиноиды в 6 и 7-й ряды. (Обычно эти элементы показывают отдельно от общей таблицы, чтобы не делать ее слишком широкой.)

Шестой ряд Периодической таблицы содержит несколько радиоактивных элементов, включая прометий, полоний, астат и радон. Астат – самый проблемный элемент этого ряда. Мы даже не знаем, как он выглядит, поскольку, по словам Лоу, «эта штука просто отказывается существовать». Астат настолько радиоактивен (его период полураспада измеряется часами), что любой крупный кусок астата быстро испарился бы от производимого им самим жара. Химики подозревают, что у этого куска была бы черная поверхность, но на самом деле этого никто не знает.

Для работы с астатом не существует инструкций по безопасности. Но если бы они существовали, там было бы снова и снова запекшейся кровью нацарапано одно только слово «НЕТ!»

Наш куб недолгое время содержал бы больше астата, чем было синтезировано за всю историю химии. Я говорю «недолго», потому что он немедленно превратился бы в столб раскаленного газа. От одного только жара все находящиеся рядом получили бы ожоги третьей степени, а здание, в котором вы все это проделываете, было бы полностью уничтожено. Облако горячего газа быстро поднялось бы в небо, излучая жар и радиацию.

Сила взрыва была бы как раз такой, чтобы привлечь к вашей лаборатории внимание максимального количества проверяющих. Будь взрыв чуть слабее, вам бы, возможно, удалось его скрыть. Будь он сильнее – и в городе не осталось бы ни одного чиновника, которому можно было бы сдать заполненные документы.

Пыль и обломки, покрытые астатом, полонием и другими радиоактивными элементами, посыпались бы из ядерного облака, сделав окружающие кварталы абсолютно непригодными для обитания.

Уровень радиации был бы крайне высоким. Как известно, для того чтобы один раз моргнуть, требуется несколько сотен миллисекунд, поэтому вы получили бы летальную дозу радиации, в буквальном смысле не успев моргнуть глазом.

Такую причину смерти называют «крайне острым радиоактивным отравлением», то есть вы бы попросту сварились.

Но седьмой ряд был бы еще хуже!

В самом низу таблицы есть некоторое количество странных элементов, которые называют трансурановыми. Долгое время у них были «имена-болванки» вроде «унунуний» и все в таком роде, но постепенно они получают настоящие названия.

Однако торопиться тут не стоит, потому что большая часть этих элементов настолько нестабильна, что их можно получить только в ускорителе частиц и они не могут существовать дольше нескольких минут. Если бы у вас вдруг оказалось 100 000 атомов ливермория (116-й элемент), спустя секунду остался бы один, и он тоже исчез бы через несколько сотен миллисекунд.

Но как это ни печально для нашего проекта, трансурановые элементы не уходят тихо и незаметно. Они исчезают в ходе радиоактивного распада. И большая их часть распадается на составляющие, которые, в свою очередь, тоже распадаются. Кубик любого элемента с достаточно большим порядковым номером распался бы за секунды, высвободив при этом огромное количество энергии.

Результат был бы не просто похож на ядерный взрыв, собственно, это и был бы ядерный взрыв. Однако, в отличие от бомбы, в нашем случае мы имели бы дело не с цепной, а с обычной реакцией. Все произошло бы мгновенно.

Поток высвободившейся энергии немедленно превратил бы вас и всю остальную таблицу в плазму. Происходящее напоминало бы взрыв ядерного заряда средней мощности, однако радиоактивное заражение было бы гораздо, гораздо хуже – на землю выпал бы настоящий салат из всего содержимого Периодической таблицы, и при этом элементы с невероятной скоростью превращались бы один в другой.

Грибовидное облако поднялось бы над городом. Верхушка его под действием собственного жара достигла бы стратосферы. Если ваша лаборатория находится в густонаселенной зоне, то число жертв в первые же секунды после взрыва было бы колоссальным, однако долговременные последствия в результате заражения оказались бы еще хуже.

Причем это было бы не какое-то там заурядное, обыденное радиоактивное заражение[20 - В этом случае мы еще могли бы пожать плечами и выкинуть его из головы.], нет, это было бы похоже на ядерную бомбу, которая продолжает и продолжает взрываться. Обломки, излучающие больше радиации, чем вся чернобыльская катастрофа, покрыли бы весь земной шар. Целые регионы были бы уничтожены, и их дезактивация заняла бы столетия.

В общем, собирать коллекцию, безусловно, очень весело, но когда речь заходит о химических элементах, не пытайтесь собрать их все.

Прыг-скок!

ВОПРОС: А что, если все жители Земли встали бы рядом и разом подпрыгнули, а потом одновременно приземлились?

    – Томас Беннет (и многие другие)

ОТВЕТ: Это один из самых популярных вопросов, которые задают на моем сайте. На него уже отвечали другие, включая Science Blogs и The Straight Dope. Они неплохо описывают кинетические аспекты ситуации. Однако это только часть истории.

Давайте приглядимся внимательнее.

Сначала предположим, что все население Земли магическим образом собралось в одном месте.

Вся эта толпа займет площадь, сопоставимую с площадью штата Род-Айленд. Впрочем, не вижу необходимости использовать обтекаемые обороты типа «площадь, сопоставимая с площадью»: это наш сценарий, и мы можем позволить себе любую степень точности. Итак, все человечество и в самом деле собралось в штате Род-Айленд.

Часы бьют полдень, и все одновременно подпрыгивают.

Как уже отмечалось в других источниках, сам прыжок не особенно повлияет на нашу планету. Земля весит больше, чем населяющие ее люди, примерно в 10 миллиардов раз. В среднем человек подпрыгивает в лучшем случае где-то на высоту полуметра. Даже если бы Земля была абсолютно жесткой и мгновенно отреагировала, мы ее оттолкнули бы меньше чем на диаметр атома.

И тут мы все снова приземляемся…

Теоретически это означает, что земной шар получит немало энергии, но она распространится по достаточно большой площади, так что все мы разве что оставим отпечатки наших ног на земле. Небольшое давление распространится по континентальной коре Северной Америки и угаснет без особых последствий.

Звук всех этих ступней, ударяющихся о землю, прозвучит как громкий хлопок, который продлится несколько секунд.

В конце концов все стихает. Проходят еще секунды.

Люди смущенно переглядываются. Кто-то кашляет.

Наконец, кто-то первым достает из кармана телефон. Через несколько секунд на свет извлекаются все пять миллиардов телефонов, существующих на Земле. Все они, даже те, которые работают в местной сети, показывают одну из версий сообщения «нет сигнала». Сотовые сети рухнули под беспрецедентной нагрузкой. Тем временем за пределами
Страница 10 из 15

Род-Айленда начинают останавливаться брошенные на произвол судьбы машины и механизмы.

Аэропорт в Уорике, Род-Айленд, способен обслужить несколько тысяч пассажиров в день. Если предположить, что отправка человечества из штата была организована заранее (то есть были в том числе организованы специальные службы, призванные обеспечить аэропорты штата топливом), то аэропорту в Уорике придется работать с эффективностью 500 % в течение многих лет, но толпа практически не уменьшится. В ней даже не появится прорех.

Если включить в работу все близлежащие аэропорты, то и тогда ситуация не слишком изменится. То же касается местной системы железных дорог. Толпы людей штурмуют грузовые корабли у причалов порта Провиденс, но запасти достаточно воды и еды для долгого путешествия весьма непросто.

Полмиллиона автомобилей штата Род-Айленд мобилизованы. Спустя мгновение автострады I-95, I-195 и I-295 превращаются в самую крупную пробку в истории планеты. Большая часть машин намертво застрянет, но нескольким счастливчикам удастся выбраться с автострады и искать объезда по проселочным дорогам.

Некоторые доберутся до Нью-Йорка или Бостона, прежде чем у них закончится топливо. Поскольку электричества к этому моменту уже, скорее всего, не будет, проще бросить свою машину и угнать другую, чем найти работающую заправку. Кто вас остановит? Все полицейские остались в Род-Айленде.

Своими краями толпа распространится в южный Массачусетс и в Коннектикут. Два случайных человека в толпе, вероятно, не будут знать языка друг друга, и практически никто не будет знаком с местностью. Штат Род-Айленд превратится в хаотическую мешанину возникающих и тут же рушащихся социальных связей. Жестокость станет обыденностью. Все голодны и все хотят пить. Магазины опустошаются. Питьевую воду достать непросто, и еще сложнее ее доставить…

В течение нескольких недель Род-Айленд превратится в кладбище миллиардов людей. Выжившие расселятся по всему миру и попытаются построить новую цивилизацию на свежих руинах предыдущей. Наш вид выжил, но популяция сильно уменьшилась. Орбита Земли осталась совершенно без изменений – планета вращается точно так же, как и до того, как весь вид Homo sapiens разом подпрыгнул.

Зато теперь мы знаем ответ.

Моль кротов

ВОПРОС: А что, если собрать моль кротов[21 - В оригинале игра слов, mole of moles. – Прим. пер.] в одном месте?

    – Шон Райс

ОТВЕТ: Последствия будут просто кошмарными.

Сначала несколько определений. Моль – это единица измерения. Однако это не стандартная единица. В сущности, это просто число – как «дюжина» или «миллиард». Если у вас есть моль чего-то, это значит, что у вас есть 602 214 129 000 000 000 000 000 этих штук (обычно записывается как 6,022?10??). Это число столь велико[22 - Один моль – это приблизительное число атомов в грамме водорода. Случайным образом это также вполне достойная оценка количества песчинок на Земле.], потому что его используют для подсчета молекул, которых обычно очень много.

Крот – это норное млекопитающее. Существует несколько видов кротов, и некоторые из них поистине ужасающие создания[23 - См., например: ru.wikipedia.org/wiki/Звездонос.].

Так как же будет выглядеть моль кротов – 602 214 129 000 000 000 000 000 зверьков разом?

Давайте начнем с самых диких допущений. Вот каков был бы приблизительно ход моих мыслей еще до того, как я взял бы в руки калькулятор. Давайте попытаемся ощутить порядок этого числа – а оно столь огромно, что по сравнению с ним числа 10, 1 и 0,1 кажутся совсем близкими одно к другому, практически равными. Давайте допустим, что они равны.

Крот достаточно маленькое животное, чтобы я мог взять его и подбросить в воздух [

]. Допустим, что предмет, который я могу подбросить, весит один фунт. Допустим далее, что один фунт равен одному килограмму. Число 602 214 129 000 000 000 000 000 выглядит в два раза длиннее триллиона, то есть это примерно триллион триллионов. Насколько я помню, триллион триллиона килограммов – это как раз столько, сколько весит наша планета.

…если кто-нибудь у вас поинтересуется: я не утверждал, что вычислять подобным образом правильно.

В общем, ясно, что мы имеем дело с кучей кротов размером примерно с планету. Это довольно грубая оценка, так как она может колебаться на несколько порядков в обе стороны.

Давайте подсчитаем точнее.

Восточноамериканский крот (Scalopus aquaticus) весит около 75 граммов, то есть моль таких кротов равен:

(6,022 ? 10??) ? 75 г ? 4,52 ? 10?? кг

Это чуть больше половины массы нашей Луны.

Млекопитающие по большей части состоят из воды. Объем килограмма воды – один литр, так что если кроты весят 4,52?10?? килограмма, то их объем примерно равен 4,52?10?? литра. Вы наверняка заметили, что мы игнорируем зазоры между кротами. Через секунду вы поймете, почему.

Кубический корень из 4,52?10?? л 3562 км, то есть речь идет о сфере с радиусом в 2210 км или кубе с гранями по 2561 км.

Если бы эти кроты оказались на поверхности Земли, они бы покрыли ее слоем толщиной 80 км, верхняя граница которого проходила там, где (прежде) заканчивалась атмосфера.

Этот душный океан сдавленного мяса уничтожил бы своей тяжестью большую часть жизни на планете, так что проделывать все это на Земле, очевидно, плохая затея.

Вместо этого давайте выведем наших кротов в межпланетное пространство. Гравитация соберет их в сферу. Мясо не очень хорошо сжимается под давлением, так что под влиянием гравитации кроты лишь слегка уменьшатся в объеме, и у нас получится кротовая планета чуть больше Луны.

Ускорение свободного падения на поверхности планеты кротов составит примерно 1/16 от земного, приблизительно как на поверхности Плутона. Планета сначала была бы слегка теплой – температура, вероятно, чуть выше комнатной, а затем гравитационное сжатие нагреет внутреннее пространство на несколько градусов.

И тут произойдет нечто весьма странное.

Планета кротов будет представлять собой гигантскую сферу из мяса. У нее много латентной энергии (в кротовой планете будет достаточно калорий, чтобы обеспечить нынешнее население Земли энергией на 30 миллиардов лет вперед). Обычно, когда органика разлагается, она выделяет большую часть этой энергии в виде тепла. Но в глубине планеты давление будет выше 100 мегапаскалей, и этого достаточно, чтобы уничтожить все бактерии и стерилизовать таким образом останки кротов, просто не останется микроорганизмов, которые могли бы разлагать ткани.

Ближе к поверхности, где давление будет ниже, разложению будет препятствовать почти полное отсутствие кислорода. Без кислорода обычное разложение невозможно, и единственные бактерии, которые смогут обеспечить этот процесс, анаэробные, то есть те, которым кислород не требуется. Анаэробное разложение не очень эффективно, но выделяет довольно много тепла. Если этому процессу ничто не будет препятствовать, он сможет раскалить планету до температуры кипения.

Но разложение вскоре начнет контролировать себя само. Мало какие бактерии могут выжить при температурах выше 60 °C, так что, когда температура повысится, бактерии начнут умирать и разложение замедлится. По всей планете трупики
Страница 11 из 15

кротов будут постепенно превращаться в кероген, органический материал, который в конце концов, будь планета еще горячее, стал бы нефтью.

Внешняя поверхность планеты будет излучать тепло в космос, остынет и заледенеет. Поскольку кроты в буквальном смысле образуют вокруг планеты меховое манто, то, когда замерзнет верхний слой, он изолирует внутреннюю часть планеты и замедлит потерю тепла. Однако потоками тепла в жидком ядре в основном будет управлять конвекция. Потоки горячего мяса и пузыри газов наподобие метана, а также воздух из легких скончавшихся кротов станут периодически пробиваться сквозь замерзшую кору кротовой планеты. В результате возникнут вулканические выбросы, настоящие гейзеры смерти, извергающие в пространство тела кротов, из которых состоит наша планета.

После нескольких веков или тысячелетий кротовая планета успокоится и охладится в достаточной степени, чтобы промерзнуть насквозь. Внутреннее ядро окажется под таким давлением, что вода в нем кристаллизируется в экзотические формы льда, такие как лед III и лед V, а в конце концов лед II и лед IX[24 - Совпадение с названием вымышленного вещества из романа Курта Воннегута «Колыбель для кошки» – случайность.].

Как ни крути, получается довольно печальная картинка. К счастью, возможен и другой вариант развития событий.

Я не знаю, каков размер всемирной популяции кротов (или в принципе объем биомассы мелких млекопитающих), но наугад прикинем, что на каждого человека на Земле приходится хотя бы пара десятков мышей, крыс, полевок и прочих мелких зверьков.

С другой стороны, в нашей галактике должны быть миллиарды планет, пригодных для обитания. При их колонизации мы бы точно завезли с собой мышей и крыс. Если бы даже одна планета из сотни была заселена мелкими грызунами в количестве, сопоставимым с земным, то спустя несколько миллионов лет совсем недолго по меркам эволюции общее количество когда-либо живших грызунов превысило бы число молекул в моле.

В общем, хотите развести моль кротов – постройте себе космический корабль…

Фен в ящике

ВОПРОС: А что, если засунуть постоянно работающий фен в воздухонепроницаемый ящик размером метр на метр на метр?

    – Dry Paratroopa

ОТВЕТ: Обычный фен потребляет 1875 Вт энергии.

Все эти 1875 Вт должны быть на что-то потрачены. Что бы ни происходило в ящике, но если внутри него потребляется 1875 Вт электроэнергии, то в конце концов они должны преобразоваться в 1875 Вт тепловой энергии.

Полезно иметь в виду, что это касается любого устройства, потребляющего энергию. Люди беспокоятся, оставив дома в розетке какое-нибудь зарядное устройство: не потребляет ли оно энергию? На этот вопрос легко ответить, потрогав прибор, включенный в сеть: если зарядка не теплая на ощупь, значит, она потребляет электричества меньше, чем на пенни в день. Маленькая зарядка для телефона, если она холодная на ощупь, потребляет меньше, чем на пенни в год. Это верно почти для любого электрического устройства[25 - Но совсем не обязательно верно для зарядных устройств, подключенных к другому прибору. Если зарядка соединена со смартфоном или ноутбуком, то электроэнергия может утекать из розетки через зарядку в устройство.].

Но вернемся к нашему ящику.

Тепло пойдет от фена в ящик. Допустим, что наш фен нельзя сломать, уничтожить или разрушить. Тогда ящик будет разогреваться изнутри до тех пор, пока его наружная поверхность не достигнет температуры примерно 60 °C. При этой температуре ящик будет излучать тепло с той же скоростью, с которой фен его выделяет, и система будет находиться в равновесии.

Он теплее моих родителей! Теперь он – мои новые родители.

Температура, при которой будет достигнуто равновесие, может быть чуть меньше, если на ящик веет ветерок или если ящик стоит на влажной или металлической поверхности, которая хорошо отводит тепло.

Если ящик сделан из металла, он будет достаточно горячим, чтобы обжечь вашу руку, если вы положите ее на ящик дольше, чем на пять секунд. Если у нас деревянный ящик, то в течение некоторого времени на нем можно будет держать руку, но есть опасность, что те его части, которые соприкасаются с феном, рано или поздно вспыхнут.

Внутри ящик будет похож на духовку. Температура, которой он достигнет, зависит от толщины стенок: чем более толстые стенки у ящика и чем хуже они проводят тепло, тем выше будет температура. Понадобятся не слишком толстые стенки, чтобы создать температуру, при которой фен сгорит.

Но предположим еще раз, что наш вообще не поддается разрушению. И если уж у нас есть такая классная штука, как неразрушимый фен, было бы просто глупо ограничивать его мощность жалкими 1875 Вт.

Если наш фен будет потреблять (и излучать) в десять раз больше – 18 750 Вт, то температура поверхности ящика превысит 200 °C (это как сковородка на слабом огне).

Интересно, насколько нам хватит шкалы на переключателе.

Пока что на шкале пугающе много места.

Теперь поверхность ящика раскалилась до 600 °C – достаточно, чтобы она начала светиться красным.

Если наш ящик сделан из алюминия, ящик начнет плавиться внутри. Если из свинца, то он начнет плавиться и снаружи. Если он из дерева, то пожар вам обеспечен. Но совершенно неважно, что происходит вокруг фена, – сам-то он у нас не поддается разрушению.

Двухмегаваттного лазера достаточно, чтобы уничтожить летящую ракету.

При температуре 1300 °C ящик станет примерно таким же горячим, как жидкая лава.

Добавим-ка еще.

Фен, возможно, не предназначался для этого…

Теперь в ящик течет 18 мегаватт энергии.

Поверхность ящика достигла температуры 2400 °C. Будь он сделан из стали, она бы уже расплавилась. Если же наш ящик изготовлен из чего-то вроде вольфрама, то он сможет протянуть чуть дольше.

Еще чуть-чуть, и придется остановиться.

Мощности 187 мегаватт достаточно, чтобы ящик раскалился добела. Немногие материалы могут уцелеть в подобных условиях, так что придется предположить, что наш ящик тоже не поддается разрушению.

Пол превратился в лаву.

К сожалению, про пол этого не скажешь.

Предположим, что прежде чем ящик прожег пол насквозь, кто-то бросит рядом с ним воздушный шарик, наполненный водой. Струя пара немедленно вышвырнет ящик через входную дверь на улицу[26 - Если мы с вами когда-нибудь окажемся запертыми в горящем здании, пожалуйста, игнорируйте мои идеи о том, как нам оттуда выбраться.].

Так, теперь у нас тут 1875 гигаватт (я соврал, сказав, что мы остановимся). Если верить фильму «Назад в будущее», сейчас у фена достаточно мощности, чтобы он смог путешествовать во времени.

Ящик светится ослепительно ярко, и подойти к нему ближе, чем на несколько сотен метров, нельзя из-за сильного жара. Вокруг разрастается озеро лавы. Все, что находится в радиусе 50–100 метров, вспыхивает. Столб жара и дыма поднимается в воздух. Периодические взрывы газа под ящиком подбрасывают его в воздух, и там, где он приземляется, начинаются новые пожары и возникают новые озера лавы.

Но мы продолжаем поворачивать переключатель.

При мощности 18,7 гигаватт
Страница 12 из 15

происходящее вокруг ящика начинает напоминать запуск космического корабля. Ящик бросают из стороны в сторону мощные восходящие потоки, которые он сам же и создает.

В 1914 году Герберт Уэллс описывал похожие приспособления в книге «Освобожденный мир»: бомба, которая вместо того, чтобы взорваться один раз, взрывается непрерывно – медленно разгорающийся ад, превращающий города в костры, которые невозможно потушить. Эта мрачная история предвосхищает развитие ядерного оружия 30 лет спустя.

Теперь наш ящик парит в воздухе. Каждый раз, вновь приближаясь к земле, он мгновенно разогревает поверхность, и стремительно расширяющийся раскаленный воздух снова подбрасывает его вверх.

Энергия 1875 тераватт сравнима с энергией взрыва склада динамита размером с большой дом, при этом склад продолжает взрываться ежесекундно.

В окрестностях ящика бушуют огненные бури – гигантские пожары, которые раздувает созданная ими же система потоков раскаленного воздуха.

Мы взяли новый рубеж: теперь фен, как ни невероятно это звучит, потребляет больше энергии, чем все остальные электроприборы на планете, вместе взятые.

Ящик, парящий высоко над поверхностью земли, каждую секунду излучает энергию, сравнимую с мощностью ядерного испытания «Тринити»[27 - Первый в мире взрыв атомной бомбы, 16 июля 1945 года, штат Нью-Мексико. – Прим. ред.].

К этому моменту дальнейший ход событий ясен. Эта штука будет летать по атмосфере, пока не уничтожит планету.

Давайте попробуем что-то изменить.

Переключим мощность на ноль, когда ящик будет пролетать над Северной Канадой. Быстро остывая, ящик начнет падать, пока в облаке пара не плюхнется в Большое Медвежье озеро.

А затем…

В данном случае это 11 петаватт.

Короткое отступление:

Официальный рекорд скорости объекта, созданного человеком, поставил зонд «Гелиос-2», который разогнался примерно до 70 км/с на орбите вокруг Солнца. Но возможно, что на самом деле этот титул по праву принадлежит металлической крышке люка весом две тонны.

Эта крышка закрывала шахту подземного ядерного полигона в Неваде во время испытаний в 1957 году. Когда в шахте был взорван заряд мощностью в одну килотонну, шахта превратилась, по сути, в ствол ядерной пушки, всей своей мощью вытолкнувшей вверх крышку люка. Камера, нацеленная на крышку, сделала только один кадр, прежде чем крышка исчезла из поля зрения, то есть она двигалась как минимум со скоростью 66 км/с. Эту крышку потом так и не нашли.

Так вот, хотя 66 км/с – это в шесть раз больше, чем вторая космическая скорость, маловероятно, чтобы крышка достигла космического пространства (что бы ни гласили по этому поводу разные популярные спекуляции). Расчеты показывают, что крышка либо была полностью уничтожена при столкновении с воздухом, либо в конце концов замедлилась и упала обратно на землю. Но если бы крышке все-таки удалось выйти с такой скоростью в космос, она смогла бы покинуть Солнечную систему лет через двадцать.

Когда мы снова включим фен, наш ящик, бултыхающийся в озере, переживет нечто аналогичное. Раскаленный пар под ним начнет расширяться, и когда ящик станет подниматься в воздух, вся поверхность озера обратится в пар. Пар, раскаленный потоком излучения до состояния плазмы, будет ускорять ящик все больше и больше.

Фотография из архива командира МКС Хэдфилда

Вместо того чтобы врезаться в плотный слой воздуха, как крышка от люка, ящик пролетит через сферу расширяющейся плазмы, которая окажет ему очень мало сопротивления. Затем выйдет из атмосферы и продолжит лететь, медленно угасая, превращаясь из второго солнца в тусклую звездочку. Да, большая часть Северо-Западных территорий Канады сгорит, зато сама Земля уцелеет!

Представьте, кое-кто все равно останется этим недоволен.

Странные (и тревожные) вопросы из папки «Входящие» сайта «А что, если?»

ВОПРОС: А что, если бы в чернобыльский реактор во время аварии сбросили антиматерию? Он прекратил бы плавиться?

    – Эй-Джей

ВОПРОС: Можно ли плакать так долго, что наступит полное обезвоживание организма?

– Карл Уилдэрмут

Последний свет человечества

ВОПРОС: А что, если бы все люди одновременно исчезли с лица Земли? Через сколько времени погаснет последний искусственный источник света?

    – Алан

ОТВЕТ: На титул «последнего огонька» будет много претендентов.

Прекрасная книга Алана Вейсмана «Земля без людей» подробно описывает, что произошло бы с домами, дорогами, небоскребами, фермами и домашними животными, если бы люди внезапно исчезли. Научно-популярный сериал «Жизнь после людей» посвящен аналогичной теме. Однако ни в книге, ни в сериале нет ответа на данный конкретный вопрос.

Начнем с очевидного: большая часть искусственных источников света долго не протянет, потому что основные электромагистрали довольно быстро разрушатся. Тепловые электростанции, которые обеспечивают большую часть электричества на Земле, требуют постоянной подачи топлива, а для этого именно люди должны принимать какие-то решения.

В отсутствие людей уменьшится потребность в энергии, но термостаты в наших квартирах будут продолжать работать. Угольные шахты и нефтяные вышки отключатся в первые же несколько часов, остальные предприятия подхватят у них эстафету. Эту ситуацию сложно решить даже с участием людей. В результате начнется нарастающая лавина отключений, которая приведет к полному параличу всех основных энергосетей.

Однако немало электричества исходит из источников, которые не зависят от основных сетей тепловых электростанций. Давайте взглянем на некоторые из них и посмотрим, когда они могут отключиться.

Дизельные генераторы

Многие люди, живущие, к примеру, на далеких островах, получают энергию при помощи дизельных генераторов. Эти устройства будут работатать, пока у них не закончится топливо, а этот срок в большинстве случаев составит от нескольких дней до нескольких месяцев.

Геотермальные электростанции

Источники электроэнергии, которым вообще не нужно топливо, поставляемое людьми, окажутся в более выигрышной ситуации. Например, геотермальные станции, использующие тепло недр планеты, некоторое время обойдутся без вмешательства людей.

Однако инструкция по эксплуатации геотермальной станции на острове Свартсенги в Исландии гласит, что раз в шесть месяцев необходимо заменять масло в трансмиссиях и смазывать все электрические моторы и сочленения. Без обслуживающего персонала, способного это сделать, некоторые станции проработают еще несколько лет, но в конце концов все механизмы разрушатся.

Ветряные турбины

Машинам, действие которых основано на силе ветра, повезет больше. Ветряки специально конструируют таким образом, чтобы они не требовали постоянного ухода, по той простой причине, что их много и залезать на них большая морока.

Некоторые ветряные турбины могут работать очень долго без вмешательства людей. Гедсерский ветряк в Дании был установлен в конце 1950-х и вырабатывал энергию в течение 11 лет без единого ремонта. Современные турбины обычно рассчитаны на 30 000 часов (3 года)
Страница 13 из 15

непрерывной работы без технического обслуживания, и, без сомнения, некоторые из них проработают еще десятилетия. И хотя бы в одном из них наверняка будет светиться хотя бы один жидкокристаллический экран на панели какого-нибудь индикатора.

Но в конце концов большая часть ветряных турбин остановится по той же причине, что и геотермальные станции: их движущиеся части заклинит без смазки.

Гидроэлектростанции

Генераторы, превращающие энергию падающей воды в электричество, продолжат какое-то время работать. Документальный сериал «Жизнь после людей» показывал интервью с оператором электростанции на плотине Гувера, который сказал, что, если бы все сотрудники ее покинули, она проработает еще несколько лет на автопилоте. Рано или поздно станция, скорее всего, станет жертвой либо засоров в системе забора воды, либо тех же механических поломок, что убьют ветряки и геотермальные станции.

Батарейки

Огоньки, использующие электричество батареек, погаснут в течение одного-двух десятилетий. Даже если батарейка не используется, она все равно постепенно разряжается. Некоторые типы батарей проживут дольше остальных, но даже те из них, которые, согласно рекламе, приспособлены для длительного хранения, смогут удерживать заряд всего десять-двадцать лет.

Есть несколько исключений. В Кларендонской библиотеке Оксфордского университета есть колокольчик на батарейке, который работает с 1840 года. Его звон практически не слышен, а на каждый удар молоточка тратится микроскопическая доля заряда. Никто толком не знает, какая именно батарея используется в колокольчике, потому что никто ни хочет разбирать его.

К сожалению, никакого света колокольчик не дает.

Ядерные реакторы

Долговечность ядерного реактора – непростой вопрос. Перейдя в режим низкой мощности, реактор может работать почти бесконечно, так высока удельная энергоемкость ядерного топлива.

Как сказано в одном интернет-комиксе:

К сожалению, на практике реакторы все равно проработают недолго. Как только хоть что-нибудь пойдет не так, ядро реактора автоматически перейдет в режим остановки. Возможно, это произойдет довольно быстро; причиной могут стать разные факторы, но наиболее вероятным будет отключение подачи энергии на реактор.

Может показаться странным, что для работы энергетической станции требуется внешний источник энергии, но каждая часть управляющей системы ядерного реактора спроектирована так, чтобы любой сбой немедленно заставил реактор остановиться. Как только извне перестанет поступать энергия – из-за того ли, что внешняя электростанция перестанет работать, или у запасных генераторов закончится топливо, – реактор отключится.

Космические зонды

Из всех творений человека наши космические аппараты могут оказаться самыми долговечными. Некоторые из них останутся на орбите на миллионы лет, но их электрическое оборудование не проживет так долго.

Пройдут столетия, и наши марсоходы будут погребены глубоко в пыли. К тому времени многие спутники рухнут обратно на землю, сойдя с орбиты. Спутники GPS на дальних орбитах протянут дольше, но со временем даже самые стабильные орбиты будут искажены под влиянием Луны и Солнца.

Многие космические аппараты работают за счет солнечных панелей, другие за счет энергии радиоактивного распада.

Марсоход «Кьюриосити», к примеру, использует энергию тепла, выделяемого кусочком плутония: этот радиоактивный материал находится в контейнере, укрепленном на вертикальном штоке.

«Кьюриосити» мог бы получать электроэнергию в течение более чем ста лет. В конце концов напряжение станет слишком слабым для его работы, но скорее всего, другие детали машины износятся еще раньше.

Итак, «Кьюриосити» выглядит многообещающе. Есть одна проблема – у него отсутствуют какие-либо осветительные приборы.

Строго говоря, у «Кьюриосити» есть фары – он использует их, чтобы подсвечивать образцы и проводить спектроскопию. Однако они включаются только тогда, когда проходят измерения. Без соответствующих инструкций (которые должны дать люди) у него не будет повода их включать.

Если на борту нет людей, космическому аппарату не очень-то нужны источники света. Зонд «Галилео», который исследовал Юпитер в 1990-е, был снабжен несколькими светодиодами в механизме устройства, записывавшего информацию о полете. Но поскольку они излучали волны в инфракрасном диапазоне, называть их «огнями» не вполне корректно. Да и в любом случае «Галилео» больше нет – как и было запланировано, зонд врезался в Юпитер в 2003 году[28 - Падение на Юпитер было необходимо, чтобы безопасно уничтожить зонд и не допустить заражения близлежащих небесных тел – например, спутника Юпитера Европы, на которой есть вода, земными бактериями.].

На других наших аппаратах тоже имеются источники света. Например, некоторые спутники GPS используют ультрафиолетовые светодиоды, чтобы контролировать статическое напряжение на поверхности оборудования, и эти элементы работают от солнечных батарей. Теоретически они могут работать столько, сколько будет светить Солнце. К сожалению, большая их часть не протянет хотя бы столько, сколько «Кьюриосити», и рано или поздно будет уничтожена в результате столкновения с космическим мусором.

Но солнечные батареи используются не только в космосе.

Солнечная энергия

Телефонные будки для экстренных вызовов, стоящие вдоль дорог в отдаленных местностях, часто работают на солнечной энергии. Обычно они оборудованы фонариком, который горит по ночам.

Так же как и ветряки, эти заброшенные в глуши устройства довольно хлопотно обслуживать, поэтому они спроектированы в расчете на длительную работу. Если солнечные панели периодически очищать от пыли и мусора, они проработают так же долго, как и электроника, которая к ним подключена.

Провода и электрические контакты рано или поздно разъест коррозия, но солнечные панели, находящиеся в сухом месте и подключенные к качественной электронике, смогут спокойно вырабатывать энергию в течение столетия, особенно если ветер или дождь время от времени будут очищать их от пыли.

Если следовать четкому определению понятия «свет», то огни, получающие питание от солнечных батарей в удаленных уголках планеты, могли бы оказаться последним уцелевшим источником искусственного света[29 - В СССР строили маяки, работавшие на радиоактивном распаде, но они уже не функционируют.].

Но есть еще один претендент, и он довольно странный.

Эффект Вавилова – Черенкова

Обычно радиоактивность невидима.

Когда-то циферблаты часов покрывали слоем радия, чтобы они светились. Однако светилась не сама радиоактивность, а фосфоресцирующая краска, наносившаяся поверх радия и испускавшая свет под действием излучения. С годами краска осыпалась, и хотя эти часы все еще радиоактивны, они уже не светятся.

Однако циферблаты – не единственный радиоактивный источник света.

Когда радиоактивные частицы проходят через материалы наподобие воды или стекла, они светятся, поскольку движутся быстрее скорости света в этой среде (но, конечно, медленнее
Страница 14 из 15

скорости света в вакууме). Это называется эффектом Вавилова – Черенкова, его пример – узнаваемое голубое свечение ядра ядерного реактора.

Некоторые из наших радиоактивных отходов, например цезий-137, расплавляют и смешивают со стеклом, затем охлаждают в плотные бруски, которые обматывают изоляцией для безопасной транспортировки и хранения.

В темноте они светятся синим.

Цезий-137 имеет период полураспада 30 лет, значит, спустя двести лет они все еще будут светиться с мощностью в 1 % от изначальной радиоактивности. Свет будет угасать со временем, но сохранит тот же синий цвет.

Вот и ответ. Спустя столетия в безлюдных бетонных бункерах будет по-прежнему светиться синим светом наш радиоактивный мусор.

Пулеметный ракетный ранец

ВОПРОС: А что, если построить ракетный ранец, использовав для этого автомат, направленный стволом вниз?

    – Роб Б.

ОТВЕТ: Я был несколько удивлен, когда узнал, что эту идею вполне можно реализовать! Но чтобы сделать все правильно, придется договариваться с русскими.

Принцип здесь довольно простой. Если вы стреляете вперед, отдача толкает вас назад. Следовательно, если стрелять вниз, отдача должна подбросить вас вверх.

Прежде всего нам придется ответить на вопрос, сможет ли автомат в принципе поднять собственный вес? Если он, допустим, весит 4 кг, а сила его отдачи – всего 3 кг, автомат не сможет оторваться от земли, не говоря уже о том, чтобы заодно поднять человека.

В инженерном деле соотношение веса и отдачи называется тяговооруженностью. Если она меньше единицы, аппарат не сможет подняться в воздух. Ракета-носитель «Сатурн-5» обладала тяговооруженностью, примерно равной 1,5.

Я не большой эксперт в области огнестрельного оружия (несмотря на то, что вырос на юге США), поэтому для того, чтобы ответить на вопрос Роба, я связался со своими приятелями в Техасе[30 - Судя по количеству боеприпасов, которые разбросаны у них по всему дому, этот штат уже превратился в постапокалиптическую зону боевых действий в духе фильма «Безумный Макс».].

Примечание:ПОЖАЛУЙСТА, не пытайтесь проделать это дома!

Как выясняется, у автомата Калашникова АК-47 тяговооруженность составляет примерно 2. Это значит, что, если поставить его автомат стволом и каким-то образом зафиксировать спусковой крючок в нажатом состоянии, автомат должен подняться в воздух, одновременно выпуская очередь.

Это верно не для всех видов автоматического оружия. Например, американской винтовке М-60, скорее всего, не хватит отдачи, чтобы оторваться от земли.

Сила тяги, которую развивает ракетный двигатель (или стреляющий автомат Калашникова), зависит, во-первых, от массы выброса, а во-вторых, от того, насколько быстро он ее выбрасывает. Тяга – это произведение двух этих величин.

Если АК-47 выпускает десять пуль весом по 8 граммов каждая за одну секунду, а начальная скорость каждой пули – 715 м/с, то тяга составит:

Так как заряженный АК-47 весит всего 4,8 кг, то он, по идее, должен взлететь и начать ускоряться вверх.

В действительности тяга окажется даже где-то на 30 % выше. Все дело в том, что автомат выбрасывает не только пули, но и горячий газ, и продукты сгорания пороха. Насколько это все повысит тягу – зависит от конкретного оружия и патронов. Общая эффективность также зависит от того, сбрасываете ли вы гильзы на землю по мере подъема, или берете их с собой. Я попросил своих техасских приятелей взвесить для моих расчетов несколько гильз. В ответ они сообщили мне, что не смогли найти весы, и тогда я подсказал им, что, учитывая объем их арсенала, им, в сущности, просто нужно найти кого-то еще, у кого весы есть[31 - В идеале кого-то с меньшим запасом боеприпасов.].

В общем, АК-47 смог бы взлететь, но у него не хватило бы тяги, чтобы поднять в воздух что-либо тяжелее белки.

Можно попробовать использовать несколько автоматов сразу. Если вниз будут стрелять два ствола, они создадут в два раза больше тяги. Если каждый автомат может поднять плюс к своей массе еще 2,5 кг, то два автомата поднимут 5 кг.

Уже понятно, к чему все идет:

В космос вы сегодня не попадете.

Если сложить вместе достаточное количество направленных вниз «калашниковых», то вес пассажира почти перестает иметь значение: он распределится по числу стволов и на каждый из последних придется ничтожная доля этого веса. Поскольку наше устройство, по сути дела, представляет собой пучок из множества отдельных автоматов, летящих параллельно, то чем больше будет количество «калашниковых», тем больше тяговооруженность нашего аппарата будет приближается к тяговооруженности отдельно взятого автомата:

Но есть одна проблема: боеприпасы.

В магазине АК-47 30 патронов. При десяти выстрелах в секунду это даст нам жалкие три секунды ускорения.

Это время можно увеличить, сделав магазин больше, но только до определенного момента: как выясняется, набивать в магазин больше 250 патронов не имеет смысла. Причина этого кроется в классической проблеме ракетостроения – топливо делает вас тяжелее.

Каждая пуля весит 8 граммов, а патрон (то есть пуля с гильзой и пороховым зарядом) весит более 16 граммов. Если в магазине будет больше 250 патронов, «калашников» станет слишком тяжелым и не сможет взлететь.

Итак, наш ранец должен состоять из большого количества АК-47 (минимум 25, но в идеале хотя бы 300), и в магазине каждого из них должно быть 250 патронов. Самые мощные варианты аппарата могли бы взлетать вверх на скорости до 100 м/с, поднимаясь на высоту полукилометра с лишним.

Вот мы и ответили на вопрос Роба: да, имея достаточное количество автоматов, взлететь можно.

Однако наша связка «калашниковых» – явно не самый совершенный реактивный ранец. Можно ли придумать что-то поинтереснее?

Мои техасские друзья предложили мне на выбор целый ряд различных пулеметов и автоматов, и я рассчитал результаты для каждого из них. Некоторые оказались весьма неплохи. Скажем, пулемет MG-42 (эта штука потяжелее, чем АК-47) может похвастаться чуть большей тяговооруженностью, чем «калашников».

Однако потом мы решили замахнуться на большее.

Авиационная пушка GAU-8 Avenger выпускает до 60 фунтовых снарядов в секунду. Отдача этой пушки эквивалентна почти 5 т, и это просто невероятно, учитывая, что ее устанавливают на штурмовике А-10 Thunderbolt, два двигателя которого вместе создают тягу всего в 4 т. Если установить на самолет две таких пушки и одновременно выстрелить из обеих вперед, одновременно включив двигатели на полную мощность, то пушки победят, и вы полетите назад.

А если установить GAU-8 на мою машину, включить нейтральную передачу и начать стрелять назад, то я нарушил бы любые ограничения скорости меньше чем за три секунды.

«Вообще мне гораздо интереснее, как вам это удалось».

Но как бы хорошо эта пушка ни подходила для ракетного ранца, русские создали установку, которая сработает еще лучше. Зенитная автоматическая пушка Грязева – Шипунова 6К30ГШ весит примерно вдвое меньше, чем GAU-8, и при этом ее скорострельность еще выше. Тяговооруженность этого орудия приближается к 40, а значит, если вы, сидя на ней верхом, направите ее вертикально вниз
Страница 15 из 15

и выстрелите, она не просто взлетит в облаке смертоносных осколков, но вы еще и испытаете бы при этом ускорение в 40g.

Это уже слишком. На самом деле, даже когда пушка надежно закреплена на самолете, отдача представляет собой проблему:

Отдача… все еще наносила ущерб самолету. Скорость стрельбы была уменьшена до 4000 выстрелов в минуту, но это не очень помогло. Посадочные фары почти всегда ломались после выстрела… Очередь из более чем 30 выстрелов почти непременно означала перегрев…

    Грег Гебель, airvectors.net (http://airvectors.net/)

Но если бы вам каким-то образом усадить на пушку пассажира, сделать всю конструкцию достаточно прочной, чтобы она выдержала отдачу, засунуть пушку в аэродинамический обтекатель и убедиться, что все это как следует охлаждается…

…то можно было бы прыгать через горы!

Равномерно вверх

ВОПРОС: А что, если вы внезапно начнете подниматься вверх со скоростью одного фута в секунду, то как именно вы умрете? Сначала замерзнете? Или сначала задохнетесь? Или как-то еще?

    – Ребекка Б.

ОТВЕТ: Ребекка, ты не забыла надеть пальто?

Один фут (30 см) в секунду – это не так уж быстро. Это значительно медленнее, чем скорость стандартного лифта. Вам потребуется 5–7 секунд, прежде чем вы подниметесь так высоко, чтобы никто из ваших друзей уже не сможет до вас дотянуться (конечно, это зависит от их роста).

Спустя 30 секунд вы будете в 30 футах (9 метрах) над землей. А если вы сейчас пролистаете книгу до страницы 200, то узнаете, что для какого-нибудь вашего друга это последний шанс бросить вам сэндвич, бутылку воды или еще что-нибудь[32 - Это не поможет вам выжить, но все же…].

Через минуту или две вы будете уже парить над верхушками деревьев. В общем и целом вам будет столь же комфортно, как и на земле. Если день ветреный, возможно, станет прохладнее, благодаря более устойчивому ветру над верхней границей леса[33 - Для этих расчетов я использую стандартный температурный профиль атмосферы, который, конечно же, может варьироваться в зависимости от места.].

Конец ознакомительного фрагмента.

Текст предоставлен ООО «ЛитРес».

Прочитайте эту книгу целиком, купив полную легальную версию (http://www.litres.ru/rendall-manro/a-chto-esli-nauchnye-otvety-na-absurdnye-gipoteticheskie-voprosy/?lfrom=931425718) на ЛитРес.

Безопасно оплатить книгу можно банковской картой Visa, MasterCard, Maestro, со счета мобильного телефона, с платежного терминала, в салоне МТС или Связной, через PayPal, WebMoney, Яндекс.Деньги, QIWI Кошелек, бонусными картами или другим удобным Вам способом.

notes

Сноски

1

В смысле, не погибнет сразу (здесь и далее примечания автора, если не оговорено иное. – Прим. ред.).

2

См. мой комикс http://what-if.xkcd.com/26 (http://what-if.xkcd.com/26), в котором объясняется, почему это происходит.

3

После того, как я впервые опубликовал все эти расчеты, физик из Массачусетского технологического института по имени Ханс Риндеркнехт рассказал мне, что он смоделировал этот сценарий на компьютерах своей лаборатории. Выяснилось, что в начале полета мяча большая часть молекул воздуха двигалась слишком быстро, чтобы это привело к синтезу: они просто пролетали бы сквозь мяч, нагревая его более медленно и равномерно, чем описано у меня.

4

А жаль, получился бы отличный энергетический напиток.

5

Также известные как делавары или ленни-ленапе.

6

Также известных как горные львы, они же кугуары, они же горные кошки.

7

Хотя вы вряд ли увидите зрелище, которое представилось европейским поселенцам: миллиарды птиц, закрывающих небо. В своей книге «1491» историк Чарльз Манн предполагает, что эти гигантские стаи могли возникнуть именно в результате колонизации и связанного с ней хаоса в экосистеме, потревоженной появлением оспы, мятлика и медоносных пчел.

8

Остров в устье р. Гудзон, самый южный округ города Нью-Йорк. – Прим. ред.

9

Город на Гудзоне к северу от Нью-Йорка. – Прим. ред.

10

Разве что рекламных щитов было поменьше.

11

Был. Но уже скоро мы с этим покончим.

12

Если что – это случайное совпадение.

13

Если вы знаете, напишите мне, пожалуйста.

14

См. http://xkcd.com/314 (http://xkcd.com/314).

15

К тому моменту, как вы это читаете, могли добавить и восьмой ряд. А если вы читаете это в 2038 году, то в таблице уже наверняка 10 рядов, но любое ее обсуждение запрещено роботами, захватившими нашу планету!

16

При условии, что все они находятся в двухатомной форме, т. е. О

, N

. Если кубик состоит из отдельных атомов, они немедленно объединятся, разогревшись при этом до нескольких тысяч градусов.

17

Дерек ведет прекрасный химический блог In the Pipeline.

18

Именно из-за этих его свойств белый фосфор используют в печально известных зажигательных бомбах и снарядах.

19

Поищите на Youtube по словам gallium infiltration, чтобы увидеть, насколько это странное зрелище.

20

В этом случае мы еще могли бы пожать плечами и выкинуть его из головы.

21

В оригинале игра слов, mole of moles. – Прим. пер.

22

Один моль – это приблизительное число атомов в грамме водорода. Случайным образом это также вполне достойная оценка количества песчинок на Земле.

23

См., например: ru.wikipedia.org/wiki/Звездонос.

24

Совпадение с названием вымышленного вещества из романа Курта Воннегута «Колыбель для кошки» – случайность.

25

Но совсем не обязательно верно для зарядных устройств, подключенных к другому прибору. Если зарядка соединена со смартфоном или ноутбуком, то электроэнергия может утекать из розетки через зарядку в устройство.

26

Если мы с вами когда-нибудь окажемся запертыми в горящем здании, пожалуйста, игнорируйте мои идеи о том, как нам оттуда выбраться.

27

Первый в мире взрыв атомной бомбы, 16 июля 1945 года, штат Нью-Мексико. – Прим. ред.

28

Падение на Юпитер было необходимо, чтобы безопасно уничтожить зонд и не допустить заражения близлежащих небесных тел – например, спутника Юпитера Европы, на которой есть вода, земными бактериями.

29

В СССР строили маяки, работавшие на радиоактивном распаде, но они уже не функционируют.

30

Судя по количеству боеприпасов, которые разбросаны у них по всему дому, этот штат уже превратился в постапокалиптическую зону боевых действий в духе фильма «Безумный Макс».

31

В идеале кого-то с меньшим запасом боеприпасов.

32

Это не поможет вам выжить, но все же…

33

Для этих расчетов я использую стандартный температурный профиль атмосферы, который, конечно же, может варьироваться в зависимости от места.

Конец ознакомительного фрагмента.

Текст предоставлен ООО «ЛитРес».

Прочитайте эту книгу целиком, купив полную легальную версию на ЛитРес.

Безопасно оплатить книгу можно банковской картой Visa, MasterCard, Maestro, со счета мобильного телефона, с платежного терминала, в салоне МТС или Связной, через PayPal, WebMoney, Яндекс.Деньги, QIWI Кошелек, бонусными картами или другим удобным Вам способом.

Здесь представлен ознакомительный фрагмент книги.

Для бесплатного чтения открыта только часть текста (ограничение правообладателя). Если книга вам понравилась, полный текст можно получить на сайте нашего партнера.

Adblock
detector